Sunteți pe pagina 1din 71

Chapter 1

Marginal Costing & Decision Making


Fixed and Variable Costs:
There are many different ways to classify the costs. For the purpose of decision making the most useful
classification of costs is according to their behaviour; i.e. Fixed, Variable and Semi Variable costs.
Costs may alter for a variety of reasons but most commonly they vary due to changes in level of activity. This
is normally defined as changes in level of production or sales.
At any given level of output, additional output can be obtained at less than proportionate cost per unit. This
is because certain items of cost will tend to remain fixed, irrespective of level of output. Therefore costs
should be analysed into variable and fixed components, for meaningful decision making.

Meaning of Marginal Costing:


Marginal Costing is a technique used for decision making. It is used to provide a basis for the interpretation
of cost data to measure the profitability of different products, processes and cost centres in the course of
decision making. It can therefore be used in conjunction with the different methods of costing such as job
costing, process costing, etc. or even with other techniques such as standard costing, or budgetary control.
Marginal Costing is a technique of decision making, which involves:

Ascertainment of total costs

Classification of total costs into fixed and variable

Use of such information for decision making

Application of Marginal Costing:


Assume the following:
S = Sales
V = Variable Costs
F = Fixed Costs
P = Profit
C = CONTRIBUTION

We know that Total Sales is equal to Total costs plus Profit.


S=V+F+P
SV=F+P = C
Under marginal costing the term Contribution refers to the difference between the Sales and Variable
Costs.

Profit Volume Ratio (P/V):


This is the ratio of Contribution with Sales, usually represented in terms of percentage. Therefore if P/V =
30%, it would mean that 30% of total sales equals to the total contribution.
Sales X P/V = Total Contribution
Once contribution is ascertained the profit can be concluded as:
Contribution Total Fixed Costs = Profit

Ideal Classes Final CA Advanced Management Accounting Volume 1


Alternatively Profit-Volume Ratio can be calculated as:
Profit-Volume Ratio

Difference in Profit
Difference in Sales

x 100

Difference in Cost
Difference in Sales

x 100

Similarly,
Variable Cost Ratio

Break Even Point:


This is a stage where the firm earn no profit neither incurs any loss. It is known as No Profit No Loss
situation. At Break Even, following are observed :

Total Costs
Fixed Costs
Profit/Loss

=
=
=

Total Revenue
Contribution
Nil

Break Even Point (in units) = Total Fixed Costs divided by Contribution per unit.
Break Even Point (in `) (Also known as Break Even Sales)
=
Total Fixed Costs divided by Contribution per unit multiplied by selling price per unit.
OR
Break Even Point (in `) (Also known as Break Even Sales)
=
Total Fixed Costs divided by P/V Ratio
Break Even Sales

Total Fixed Costs


P/V

x 100

Required Sales to earn Desired Profit:

Required Sales
=
(To earn Desired Profit)

Total Fixed Costs + Desired Profit


P/V

x 100

Margin of Safety:
This is the difference between the actual sales and Break Even Sales, usually in terms of amount, but
sometimes represented even in terms of number of units. In other words it represents the excess of Sales
over the Break Even Point.

Margin of Safety %

Margin of Safety
Actual Sales

x 100

Ideal Classes Final CA Advanced Management Accounting Volume 1

Section 1 CVP Analysis


Q.1.

Consider the following information:


`
20
30
10
60

Direct material cost per unit


Direct labour cost per unit
Variable overheads per unit
Variable cost per unit
Total fixed overheads ` 4,00,000
Selling price per unit ` 125
Normal capacity 10,000 units
Practical capacity 12,500 units
There is an offer to supply additional 2,500 units at a price of ` 80 per unit. This sale will not affect
the present market of the product.
You are required to calculate the following:
1.
Full cost per unit at present
2.
Amount of profit earned at present
3.
Revised amount of cost per unit, when production increases from 10,000 units to 12,500 units
on accepting the offer.
Should the offer be accepted? Indicate the financial implication of accepting the offer.
Q.2.

(Behaviour of Costs)
Consider the data given below:
Particulars
Year 1
Year 2
Output in units
10,000
12,000
Overheads (1) `
3,28,000
3,28,000
Overheads (2) `
3,00,000
3,60,000
Overheads (3) `
3,18,000
3,80,160
Overheads (4) `
3,28,000
3,53,600
Overheads (5) `
3,26,500
3,51,800
Overheads (6) `
3,28,000
3,93,600
Overheads (7) `
3,29,500
3,81,400
You are required to identify the behavior of overheads and classify these into:

Fixed Overheads

Variable Overheads

Semi Variable Overheads


If the overheads are identified as Semi Variable, then segregate these into their:

Fixed Portion (in terms of per annum)

Variable Portion (in terms of per unit)

Q. 3.

Selling price per unit of the product is ` 320 and its variable cost per unit is ` 192. The company
earns a net profit at present @ ` 30 per unit at a present sales volume of 8,000 units. You are
required to calculate the following:
1. Profit Volume Ratio
2. Break Even Point (BEP) in units
3. Break Even Level of sales
4. Margin of safety in terms of amount and percentage
5. Required Sales to earn a desired profit of ` 7,52,000
6. Required Sales to earn a desired profit of 26% on sales
7. Required Sales to earn a desired profit of 25% on total cost

Ideal Classes Final CA Advanced Management Accounting Volume 1


Q. 4.

X Ltd manufactures and sells 3 products, details of which are mentioned below:
Selling Price
Variable Cost
Percentage of
Per unit (`)
Per unit (`)
Total units produced and sold
A
100
70
30%
B
80
60
30%
C
50
37.5
40%
The total fixed cost for the year is ` 16,80,000.
Find the overall BEP in units maintaining the same product mix as above and also calculate the
number of units of each product as part of the total BEP.

Q.5.

The budgeted results of A Ltd. are as under:


Products
Sales value
(`)
X
2,50,000
Y
4,00,000
Z
6,00,000
Total
12,50,000

P/V ratio
(%)
50
40
30

Sales mix
(%)
20
32
48
100

Fixed overheads for the period are ` 5,02,200.


The management is worried about the results. You are required to prepare:
1. A statement showing the amount of loss, if any, being incurred at present and recommend a
change in the sale value of each product as well as in the total sales value maintaining the same
sales-mix, which will eliminate the said loss.
2. Recommend the additional sales of any individual product to recover the loss.
(May 1999)
Q. 6.

Q.7.

Calculate the Break Even Sales from the following details:


Particulars
Year 1
Sales (`)
30.0 Lakhs
Total Costs (`)
27.0 Lakhs

Year 2
36.0 Lakhs
31.8 Lakhs

(Multiple BEP)
You have been approached by a friend who is seeking your advice as to whether he should give up
his job as an engineer, with a current salary of ` 14,800 per month and go into business on his own,
assembling and selling a component which he has invented. He can procure the parts required to
manufacture the component from a supplier.
It is very difficult to forecast the sales potential of the component, but after some research your
friend has estimated the sales as follows :
I.
Between 600 to 900 components per month at a selling price of ` 250 per component.
II.
Between 901 to 1,250 components per month at a selling price of ` 220 per component for
the entire lot.
The cost of the parts required would be ` 140 for each completed component. However if more
than 1,000 components are produced in each month, a discount of 5% would be received from the
supplier of parts on all purchases.
Assembly costs would be ` 60,000 per month upto 750 components. Beyond this level of activity
assembly costs would increase to ` 70,000 per month.
Your friend has already spent ` 30,000 on development, which he would write-off over the first five
years of the venture.
Required:

Ideal Classes Final CA Advanced Management Accounting Volume 1

Q. 8.

Q. 9.

1. Calculate for each of the possible sales levels at which your friend could expect to benefit by
going into the venture on his own.
2. Calculate the break-even point of the venture for each of the selling price.
3. Advise your friend as to the viability of the venture.
(Nov 1996)
(Indifference Point)
The following are cost data for three alternative way of processing the clerical work for cases
brought before the LC Court System:
(`)
Particulars
A
B
C
Manual
Semi automatic
Fully automatic
Fixed costs :
Occupancy
15,000
15,000
15,000
Maintenance contract
0
5,000
10,000
Equipment lease
0
25,000
1,00,000
15,000
45,000
1,25,000
Variable costs (per report)
Supplies
40
80
20
Labour
5 hrs X 40
1 hr X 60
0.25 hr X 80
or 200
or 60
or 20
240
140
40
Required:
1. Calculate cost indifference points. Interpret your results.
2. If the present case load is 600 cases and it is expected to go up to 850 cases in near future, which
method is most appropriate on cost considerations?
(May 2001)
(Absorption vs. Marginal Costing)
Consider the data given below for the first two years of operations of NJ Ltd.:

Particulars
Budgeted Fixed Overheads (`)
Actual Fixed Overheads (`)
Budgeted Production (units)
Actual Production (units)
Actual Sale (units)
Selling Price per unit (`)
Variable Cost per unit (`)

Year 1

Year 2

13,20,000
13,60,000
100,000
120,000
108,000
50
30

1,350,000
1,480,000
125,000
120,000
132,000
50
30

You are required to calculate profit of the two years as per:


1.
Marginal Costing
2.
Absorption Costing
Q.10.

(Multiple BEP)
Navbharat College, Bombay has six sections of B.Com, and two sections of M.Com with 40 and 30
students per section respectively. The college plans one day pleasure trip around the city for the
students once in an academic session during winter break to visit park, zoo, planetarium and
aquarium.
A transporter used to provide the required number of buses at a flat rate of ` 700 per bus for the
aforesaid purpose. In addition, a special permit fee of ` 50 per bus is required to be deposited with

Ideal Classes Final CA Advanced Management Accounting Volume 1


city Municipal Corporation. Each bus is 52 seater. Two seats are reserved for teachers who
accompany in each bus. Each teacher is paid daily allowance of ` 100 for the day. No other costs in
respect of teachers are relevant to the trip. The approved caterers of the college supply breakfast,
lunch and afternoon tea respectively at ` 7, ` 30 and ` 3 per student.
No entrance fee is charged at the park. Entrance fees come to ` 5 per student both for the zoo and
the aquarium. As regards planetarium the authorities charge block entrance fee as under for group
of students of educational institutions depending upon the number of student in group:
Number of Students in a Group

Block Entrance Fee


`
200
300
450

Up to 100
101-200
201 & above

Cost of prizes to be awarded to the winners in different games being arranged in the park depends
upon the strength of students in a trip. Costs of prizes to be distributed are:
Number of Students in a Trip

Cost of Prizes
`
900
1,050
1,200
1,300
1,400
1,500

Upto 50
51-125
126-150
151-200
201-250
251 & above

To meet the above costs the college collects ` 65 from each student who wish to join the trip. The
college releases subsidy of ` 10 per student in the trip towards it.
You are required to:
1.
Prepare a tabulated statement showing total costs at the levels of 60, 120, 180, 240 and 300
students indicating each item of cost.
2.
Compute average cost per student at each of the above levels.
3.
Calculate the number of students to break even for the trip as the college suffered loss during
the previous year despite 72% of the students having joined the trip.

Practice Section
Q.11.

The following information of a company is available for the year 2006:

`
Sales
Raw materials
Direct wages
Variable and fixed OH
Profit
Units sold

40,000
20,000
6,000
10,000
4,000
200 Nos.

In the year 2007, wages rate will increase by 50% and fixed cost will decrease by ` 600. If 300 units
are sold in 2007, the total fixed and variable OH will be 11,400. How many units should be sold in
2007, so that the same amount of profit per unit as in year 2006 may be earned?
(May 07, 4 Marks)

Ideal Classes Final CA Advanced Management Accounting Volume 1


Q.12. A company makes 1,500 units of a product for which the profitability statement is given below:
`
1,20,000

Sales
Direct materials
30,000
Direct labour
36,000
Variable OH
15,000
Subtotal variable cost
81,000
Fixed cost
16,800
Total cost
97,800
Profit
22,200
After the first 500 units of production, the company has to pay a premium of ` 6 per unit towards
overtime labour. The premium so paid has been included in the direct labour cost of ` 36,000 given
above. You are required to compute the Break-even point.
(May 07, 6 Marks)
Q.13. X Ltd. manufactures a semiconductor for which the cost and price structure is given below:

` per unit
Selling price
500
Direct material
150
Direct labour
100
Variable overhead
50
Fixed cost = ` 2 lakhs.
The product is manufactured by a machine, whose spare part costing ` 2,000 needs replacement
after every 100 pieces of output. This is in addition to the above costs. Assume that no defectives are
produced and that the spare part is readily available in the market at all times at ` 2,000.
1. Prepare the profitability statement for production levels of 2,000 units and 3,000 units, when
fixed cost = ` 1 lakhs.
2. What is the break-even point (BEP) for the above data?
3. Comment on the BEP, if the fixed cost can be reduced to ` 1,80,000 from the existing level of 2
lakhs.
(Nov 06, 14 Marks)
Q.14. The working results of a Software Company for two corresponding years are shown below:

Sales (A)
Cost of Sales:
Direct materials
Direct wages and variable overheads
Fixed overheads
Total (B)
Profit (A B)

Year 2005
600

Amount (` in lakhs)
Year 2006
770

300
180
80
560
40

324
206
150
680
90

In year 2006, there has been an increase in the selling price by 10 per cent. Following are the details
of material consumption and utilization off direct labour hours during the two years:
Year 2005
5,00,000
75,00,000

Direct material consumption (M. tons)


Direct labour hours

Year 2006
5,40,000
80,00,000

Ideal Classes Final CA Advanced Management Accounting Volume 1


Required:
1. Taking year 2005 as base year, analyse the variances of year 2006 and also workout the amount
which each variance has contributed to change in profit.
2. Find out the breakeven sales for both years.
3. Calculate the percentage increase in selling price in the year 2006 that would be needed over
the sale value of year 2006 to earn margin of safety of 45 per cent.
(Nov 07, 19 Marks)
Q.15. A Ltd. Makes and sells a single product. The companys trading results for the year are:

Sales
Direct materials
Direct labour
Overheads
Profits

Figs. ` 000 (Year 2007)


3,000
900
600
900
2,400
600

For the year 2008, the following are expected:


1. Reduction in the selling price by 10%.
2. Increase in the quantity sold by 50%.
3. Inflation of direct material cost by 8%.
4. Price inflation in variable overhead by 6%.
5. Reduction of fixed overhead expenses by 25%.
It is also known that:
(a) In 2006, overhead expenditure totalled to ` 8,00,000.
(b) Total overhead cost inflation for 2007 has been 5% more than 2006.
(c) Production and sales volumes have been 25% higher in 2007 than in 2006.
The high-low method is being used by the company to estimate overhead expenditure.
You are required to:
1. Prepare a statement showing the estimated trading results for 2008.
2. Calculate the Break-even point for 2007 and 2008.
3. Comment on the BEP and profits of the years 2007 and 2008
(May 08, 12 Marks)
Q.16.

A single product manufacturing company has an installed capacity of 3,00,000 units per annum. The
normal capacity utilization of the company is 90%. The company has prepared the following budget
for a year:
Variable costs:
Factory costs
` 33 per unit
Selling and Administration costs
` 9 per unit
Fixed costs:
Factory costs
` 21,60,000
Selling and Administration costs
` 7,56,000
Selling Price
Selling price per unit
` 60
The actual production, sales, price and cost data relating to the year under review are as given
below:
Production
2,40,000 units
Sales
2,25,000 units
Finished goods stock in the beginning of the year:
15,000 units
Actual factory variable costs exceeded the budget by
` 1,20,000

Ideal Classes Final CA Advanced Management Accounting Volume 1


Required:
1. Calculate the budgeted profit and break-even point in units.
2. What increase in selling price was necessary during the year under review to maintain the
budgeted profit?
3. Prepare statements showing the actual profit during the year under review by using (1)
absorption costing method and (2) marginal costing method.
(Nov 08, 11 Marks)
Q.17.

A company uses absorption costing system based on standard costs. The total variable
manufacturing cost is ` 6 per unit. The standard production rate is 10 units per machine hour. Total
budgeted and actual fixed production overhead costs are ` 8,40,000. Fixed production overhead is
allocated at ` 14 per machine hour. Assume this same standard for the last year and current year.
Selling price is ` 10 per unit.
Variable selling overheads are ` 2 per unit and fixed selling costs are ` 2,40,000. Assume that there
are no price, spending or efficiency variances. Beginning inventory was 30,000 units and ending
inventory was 40,000 units.

Q.18.

1. Compute the break-even point under absorption costing, assuming that there will be an under
absorption of overhead and that production variance is written off at year end as adjustment to
cost of goods sold.
2. Compute the break-even point under marginal costing.
3. Assuming that sales were at break-even level computed under (2) above, and that production
variance is written off at the year end as adjustment to cost of goods sold, and that stock levels
were as given above, find the profit under absorption costing. (detailed cost statement not
essential)
(May 2010, 19 Marks)
The following information is given by Z Ltd:
Margin of safety
` 1,87,500
Total cost
` 1,93,750
Margin of safety
7,500 units
Break-even sales
2,500 units
Required:
Calculate Profit, P/V Ratio, BEP Sales (in `) and Fixed Cost.
(Nov 2010, 4 Marks)

Home Work Section


Q.19.

If fixed costs are ` 4,000, variable costs ` 32,000 and breakeven point ` 20,000 find out:
1. Profit volume ratio;
2. Sales;
3. Net Profit;
4. Margin of safety

Q.20.

If fixed costs are ` 24,000, Margin of safety ` 40,000 and Break even ` 80,000 find out:
1. Sales;
2. Profit volume ratio;
3. Net Profit and
4. Variable costs.

Ideal Classes Final CA Advanced Management Accounting Volume 1


Q.21.

Veejay Ltd. makes and sells two products, Vee and Jay. The budgeted selling price of Vee is ` 1,800
p.u. and that of Jay is ` 2,160 p.u. Variable costs associated with producing and selling the Vee are `
900 p.u. and with Jay ` 1,800 p.u. Annual fixed production and selling cost of Veejay Ltd. are `
88,000.
The company has two production / sales options. The Vee and Jay can be sold either in the ratio of
two Vees to three Jays or in the ratio of one Vee to two Jays. What will be the optimal mix and why?

Q. 22. (a)

Calcutta Company Ltd. manufactures and sells four types of products under the brand names
Ace, Utility, Luxury and Supreme. The sales mix in value comprises the following:
Brand
Ace
Utility
Luxury
Supreme
Total

(b)

Percentage
331/3 %
412/3 %
162/3 %
81/3 %
100 %

The total budgeted sales (100%) are ` 6, 00,000 per month.


The operating costs are:
Ace
60% of selling price
Utility
68% of selling price
Luxury
80% of selling price
Supreme
40% of selling price
The fixed costs are ` 1, 59,000 per month. Calculate the break-even point for the products on
an overall basis.
It has been proposed to change the sales mix as follows, the total sales per month remaining
` 6,00,000.
Brand
Ace
Utility
Luxury
Supreme

Percentage
25%
40%
30%
5%
100%

Assuming that this proposal is implemented, calculate the new break-even point.
Q. 23. (Multiple BEP)
Satish Enterprise are leading exports of Kids Toys, J Ltd. of U.S.A have approached Satish Enterprises
for Exporting a special toy named Jumping Monkey. The order will be valid for next three years at
3,000 toys per month. The export price of the toy will be $4. Cost data per toy is as follows:
(`)
Material
60
Labour
25
Variable Overheads
20
Primary packing of the toy
15
The toys will be packed in lot of 50 each. For this purpose a special box, which will contain the 50
toys will have to be purchased, cost being ` 400 per box.
Satish Enterprise will also have to import a special machine for making the toys. The cost of the
machine is ` 24,00,000 and duty there on will be at 12%. The machine will have an effective life of 3
years and depreciation is to be charged on straight line method. Apart from depreciation annual
fixed overheads is estimated at ` 4,00,000 for the first year with 6% increase in the second year.
Fixed overheads are incurred uniformly over the year.
Assuming the average conversion rate to be ` 50 per $, you are required to:

10

Ideal Classes Final CA Advanced Management Accounting Volume 1


1. Prepare a monthly and yearly profitability statements for the first year and second year
assuming the production at 3,000 toys per month.
2. Compute a monthly and yearly break even units in respect of the first year.
(Nov 1999)
Q.24.

A company produces and sells four types of dolls for children. It also produces and sells a set of dress
kit for the dolls. The company has worked out the following estimates for the next year:
Doll

A
B
C
D
Dress Kit

Estimated
Demand (Units)
50,000
40,000
35,000
30,000
2,00,000

Standard
Materials Cost
Per unit (`)
20
25
32
50
15

Standard
Labour Cost
Per unit (`)
15
15
18
20
5

Estimated
Sales Price
Per unit (`)
60
80
100
120
50

To encourage the sales of dress kits to 2,00,000 units, as above, a discount of 20% in its price is
offered if it were to be purchased along with the doll. It is expected that all the customers buying
dolls will also buy the dress kit.
The companys factory has effective capacity of 2,00,000 labour hours per annum on a single shift
basis and it produces all the products on that basis. The labour hour rate is ` 15. Overtime of a
labour has to be paid at double the normal rate. Variable cost works out to 40% of direct labour cost.
Fixed costs are ` 30 lakhs per annum. There will be no inventory at the end of the year.
You are to draw a conservative estimate of the years profitability.
(May 1998)
Q.25.

Following are the details available from the records of two companies in similar business line:
(All figures in ` Lakhs)
Zero Ltd
Hero Ltd
Sales
20
20
Less:
1. Variable Costs
(12)
(8)
2. Fixed Costs
(4)
(8)
Profit
4
4
You are required to comment on the performance of the above two companies as to which is better
and why?

Q.26.

(Indifference Point)
M Companys central services department is evaluating new copying machines to replace the firms
current copier, which is worn out. The analysis of alternative machines has been narrowed to three
and the estimated costs of operating them are shown below:
(`)
Machine A
60
80
30,000

Cost per 100 copies


Machine B
40
30
58,000

Machine C
20
20
1,00,000

Materials costs
labour cost
Annual lease cost
Required:
1. Compute the cost indifference points for the three alternatives.
2. What do the cost indifference points suggest as a course of action in this regard?
If the management expects to need 87,000 copies next year, which copier would be most
economical?

11

Ideal Classes Final CA Advanced Management Accounting Volume 1


Q.27.

(Absorption vs. Marginal Costing)


Topless Products Ltd. has several product lines with a sales manager in charge of each product line
and he is paid a bonus based on the net income generated by his product line.
In analyzing performance of one product line, the General Sales Manager noted that the sales
declined from ` 8,00,000 to ` 6,00,000 for the current year. However, the product line Manager
received a larger bonus than last year because net income increased from ` 90,000 last year to `
1,20,000 for the current year. The General Manager wonders how the product line manager is
entitled to a bonus with a decline in sales. He also wants to know how net income increased, when
sales declined.
As a Cost Accountant you are required to prepare the income statements, based on which the bonus
was paid. Explain with supporting figures why net income increased when sales declined. What do
you think of the present method of paying the bonus? Can you suggest some other method?
The data given in support for the bonus payment are:
Particulars
Units sold @ ` 20
Standard variable cost of production per unit (`)
Fixed factory overhead cost
(`)
Selling & distribution expenses (fixed)
(`)
Standard Fixed factory overhead per unit
(`)
Units produced
Opening finished goods inventory (Units)

Year 2
30,000
8
2,00,000
1,40,000
5
50,000

Year 1
40,000
8
2,00,000
1,40,000
5
30,000
10,000

All factory overhead variances are written off to cost of goods sold.
(May 1990)
Q.28.

(Absorption vs. Marginal Costing)


Mr. Raghavan is quite displeased and frustrated as despite his and his staffs best efforts, although
the sales are increasing; the profits are declining over the last three years. He supplies you with the
following information and asks your help to clear the picture:
(` '000's)
Particulars
2002
2003
2004
Sales (At ` 20 per unit)
1,000
1,100
1,200
Cost of production:Variable
260
240
160
Fixed (applied)
390
360
240
Opening inventory (Added)
50
200
250
Closing inventory (Deducted)
200
250
50
500
550
600
Adjustment for overheads applied
(-) 30
(+)120
Actual cost of goods sold
470
550
720
Gross Profit
530
550
480
Less: Selling expenses (semi-variables)
490
530
570
Net Profit (+) / Loss (-)
(+) 40
(+)20
(-)90
Actual production for the last three years was 65,000, 60,000 and 40,000 units respectively. 5,000
units were in stock at the beginning of 2002. Fixed manufacturing overheads are applied to
production based on planned activity of 60,000 units every year. Actual overheads were ` 10,80,000
for past three-year period and were evenly incurred.
(May 1994)

12

Ideal Classes Final CA Advanced Management Accounting Volume 1


Q.29.

(Absorption vs. Marginal Costing)


Wonder Ltd. manufactures a single product, Zest. The following figures relate to Zest for a one-year
period:
Activity Level
50%
100%
Sales and production (units)
400
800
` Lakhs
` Lakhs
Sales
8
16
Production costs:
Variable
3.2
6.4
Fixed
1.6
1.6
Selling and administration costs:
Variable
1.6
3.2
Fixed
2.4
2.4
The normal level of activity for the year is 800 units. Fixed costs are incurred evenly throughout the
year, & actual fixed costs are the same as budgeted. There were no stocks of Zest at the beginning of
the year.
In the first quarter, 220 units were produced and 160 units were sold.
Required:
1. What would be the fixed production costs absorbed by Zest if absorption costing is used?
2. What would be the under/over-recovery of overheads during the period?
3. What would be the profit using absorption costing?
4. What would be the profit using marginal costing?
5. Why is there a difference between the answers to (3) and (4)?

13

Ideal Classes Final CA Advanced Management Accounting Volume 1

Section 2 Relevant Cost Approach


Q. 30. A new job requires 10,000 kgs of a material. 21,000 kgs of the same material is already in stock. This
material is regularly used by the company. Other information:
Actual purchase price of material in stock is ` 20 per kg.
Current Market Price is ` 25 per kg.
Resale value (NRV) ` 16 per kg.
You are required to determine the relevant cost of material for this new job.
Q. 31. A new job requires 10,000 kgs of a material. 6,000 kgs of the same material is already in stock. This
material is regularly used by the company. Other information:
Actual purchase price of material in stock is ` 20 per kg.
Current Market Price is ` 25 per kg.
Resale value (NRV) ` 16 per kg.
You are required to determine the relevant cost of material for this new job.
Q. 32. A new job requires 10,000 kgs of a material. 10,000 kgs of the same material is already in stock. This
material has no other use. Other information:
Actual purchase price of material in stock is ` 20 per kg.
Current Market Price is ` 25 per kg.
Resale value (NRV) ` 16 per kg.
You are required to determine the relevant cost of material for this new job.
Q. 33. A new job requires 10,000 kgs of a material. 6,000 kgs of the same material is already in stock. This
material has no other use. Other information:
Actual purchase price of material in stock is ` 20 per kg.
Current Market Price is ` 25 per kg.
Resale value (NRV) ` 16 per kg.
You are required to determine the relevant cost of material for this new job.
Q. 34. A new job requires 10,000 kgs of a material. 10,000 kgs of the same material is already in stock. This
material has no other use. Other information:
Actual purchase price of material in stock is ` 20 per kg.
Current Market Price is ` 25 per kg.
Resale value (NRV) = Nil.
You are required to determine the relevant cost of material for this new job.
Q. 35. A new job requires 10,000 kgs of a material. 10,000 kgs of the same material is already in stock. This
material has one use other than this new offer. The material can be substituted for another material
which otherwise would have been purchased for ` 1,00,000. This substitution requires adaption of
this material, which will cost ` 12,000.
Other information:
Actual purchase price of material in stock is ` 20 per kg.
Current Market Price is ` 25 per kg.
Resale value (NRV) = Nil.
You are required to determine the relevant cost of material for this new job.
Q. 36. A new job requires 10,000 kgs of a material. 7,000 kgs of the same material is already in stock. This
material has one use other than this new offer. The material can be substituted for another Material
X which is purchased at ` 9 per kg. Each kg of this material is equivalent to 2 kgs of material X. To
make this material suitable for substitution, the adaption cost will be ` 20,000. Other information:

14

Ideal Classes Final CA Advanced Management Accounting Volume 1


Actual purchase price of material in stock is ` 20 per kg.
Current Market Price is ` 25 per kg.
Resale value (NRV) = ` 16 per kg.
You are required to determine the relevant cost of material for this new job.
Q. 37. A new job requires 10,000 kgs of a material. This material is in short supply and no quantity can be
purchased from market at present. However, 10,000 kgs of material is already in stock but for
another job. The only way to make this material available for the new job is to divert the material
from the existing job to new job. The existing job for which this material was purchased provides the
following contribution for each kg of material:

` per kg
Selling Price
100
Less: Material Cost
20
Less: Processing Cost
68
Contribution
12
Other information:
Actual purchase price of material in stock is ` 20 per kg.
Current Market Price = ???.
Resale value (NRV) = ` 16 per kg.
You are required to determine the relevant cost of material for this new job.

Q. 38. Required Quantity of Material


=
10,000 Kgs
Book Value of existing stock
(Actual Purchase Price)
=
` 50 per kg
Current purchase price (Replacement price)
=
` 60 per kg
Resale Value (NRV)
=
` 45 per kg
What shall be the relevant cost of the required 10,000 kgs of the material?
Case 1:
Material is in regular use and 24,000 Kgs are available in current stock.
Case 2:
Material is in regular use and 6,000 Kgs are available in current stock.
Case 3:
Material is in regular use but supply is restricted. Say only 10,000 Kgs are available in stock, which
can also be used for another offer where net contribution from the offer (after considering the
purchase cost of material) is ` 8,00,000 for 10,000 Kgs use of material. No additional supplies can be
obtained from outside market.
Case 4:
10,000 kgs of the material is in stock and it is idle/non moving in nature (Material has no other use).
Case 5:
6,000 kgs of the material is in stock and it is idle/non moving in nature (Material has no other use).
Case 6:
10,000 kgs of the material is in stock and it is idle/non moving in nature (Material has no other use).
There is no resale value of the material.
Case 7:
6,000 kgs of the material is in stock and it is idle/non moving in nature (Material has no other use).
There is no resale value of the material.

15

Ideal Classes Final CA Advanced Management Accounting Volume 1


Case 8:
10,000 kgs of the material is in stock and is idle/non moving in nature. (Material has no other use).
There is no resale value of the material. The material is also toxic in nature. So, management wants
to dispose it off, if not used for this offer and the cost for disposal would be ` 12,000.
Q.39.

A Limited has been offered a contract that, if accepted, would significantly increase next years
activity level. The contract requires the production of 20,000 kgs of product X and specifies a
contract price of ` 1,000 per kg. The resources required in the production of each kg of X include the
following:
Resources per kg of X
Labour :
Grade 1
2 hours
Grade 2
6 hours
Materials :
A
2 units
B
1 litre
Grade I labour is highly skilled and although currently under-utilised in the firm, it is As policy to
continue to pay Grade I labour in full. Acceptance of the contract would reduce the idle time of
Grade I labour. Idle time payments are treated as non-production overheads. Grade 2 is unskilled
with a high turnover, and may be considered as variable cost. The cost to A for each type of labour
are: Grade I ` 40 per hour, Grade 2 ` 20 per hour.
The materials required to fulfill the contract would be drawn from the materials already in stock.
Material A is widely used within the firm and any usage for the contract will necessitate
replacement. Material B was purchased to fulfill an expected order that was not received. If material
B is not used for the contract, it will be sold.
For accounting purposes FIFO is used. The various values and costs for A and B are as follows:
(` Per unit )
Particulars
Book value
Replacement cost
Net realizable value

A
80
100
90

B
300
320
250

A single recovery rate for fixed factory overheads is used throughout the firm, even though some of
these costs could be attributed to a particular product or department. The overhead is recovered by
applying a pre-determined rate per productive labour hour. Initial estimates of next years activity,
which exclude the current contract, show fixed production overhead of ` 60,00,000 and production
labour hours of 3,00,000. Acceptance of the contract would increase fixed production overheads by
` 22,80,000. Variable production overheads are accurately estimated at ` 30 per productive labour
hour.
Acceptance of the contract would encroach on the resources used to produce and sale another
product Y, which is also made by A Ltd. It is estimated that the sale of Y would then decrease by
5,000 units in the next year. However, this reduction in sale of Y would enable attributable fixed
overhead of ` 5,80,000 to be avoided. Information on Y is as follows:
(Per unit)
Selling price
` 700
Labour Grade 2
4 hours
Materials relevant variable costs
` 120
Required:
Advise A Ltd on the desirability of the acceptance of the contract purely on economic considerations.
Show your calculations
(May 2002)

16

Ideal Classes Final CA Advanced Management Accounting Volume 1


Q.40.

Jobanputra Ltd. manufactures a wide range of kitchen utilities. The company is considering whether
to add a further product the Niks to the range. The demand for the Niks will be 60,000 units that
could be sold at ` 285 per unit.
The following information is available regarding the cost of manufacturing Niks:
Direct Materials:
Raw
Material

Quantity
required
per unit
of Niks

Current
stock
level

A
B
C
D
E

1
2
0.5
1
1.5

60,000
60,000
60,000
80,000
75,000

Costs per unit of raw material


Original
Current
cost
Replacement
cost
`
`
2
2.4
6
5
9
10
6.5
----12
20

Material A is an obsolete material. It can only be used on another product, the material for which is
available at ` 95,000. For this purpose, Material A requires some adaptation to be used; adaptation
would cost ` 15,000.
Material B was ordered for some other product which is no longer required. It now has a residual
value of ` 2,00,000.
Material C is already in stock and is widely used within the firm. Although present stocks together
with orders already planned will be sufficient to facilitate normal activity and extra material used by
adopting this alternative will necessitate such materials being replaced immediately.
Material D is also in stock but it is unlikely that any additional supplies can be obtained for some
considerable time. At the present time Material D is normally used in the production of Product
Ash, which sells at ` 90 per unit and incurs total variable cost (excluding Material D) of ` 50 per
unit. Each unit of Product Ash uses four units of Material D.
Material E was ordered for some other product which is no longer required. It now has a residual
value of ` 8,00,000. Alternatively, existing stock of material E can be used in another job as
substitute for material X which currently cost ` 8 per unit (of which the company has no units in
stock at the moment). Each unit of material E is equivalent to 2 units of Material X. For this purpose,
Material E requires some adaptation to be used; adaptation would cost ` 4,25,000.
Direct Labour:
The following categories of labour shall be required:
Kalakaari labour
paid @ ` 20 per hour
Mehnati labour
paid @ ` 16 per hour
Manmaani labour
paid @ ` 12 per hour
Ghulaami labour
paid @ ` 10 per hour
Each unit of Niks requires:
2 Hrs of Kalakaari labour
3 Hrs of Mehnati labour
5 Hrs of Manmaani labour
8 Hrs of Ghulaami labour
Kalakaari labour is in short supply and cannot be increased significantly in the short term; this labour
is presently engaged in meeting the demand for product Jumbo, which requires 4 hours of
Kalakaari labour. The contribution from the sale of one unit of product Jumbo is ` 60, as shown
below:

17

Ideal Classes Final CA Advanced Management Accounting Volume 1


` p.u.
Selling price
Direct Material
Direct Labour (Kalakaari)
(4 Hrs @ ` 20 per hour)
Variable Overheads

320
60
80

Contribution

60

120

This diversion would cost additional incentive of ` 1.00 per hour.


Mehnati labour is part of the permanent labour force, and is idle. However the idle time available at
present is only 70,000 hours and this will not be sufficient for Production of Niks. For the
remaining requirement, it would require diverting Mehnati labour from another job, on which they
are earning a contribution surplus of ` 10 per labour hour, as shown below:
` p.u.
Selling price
90
Direct Material
40
Direct Labour (Mehnati)
16
(1 Hr @ ` 16 per hour)
Variable Overheads
24
Contribution
10
Manmaani labour is employed on a casual basis and sufficient labour can be acquired to exactly
meet the production requirements.
Ghulaami labour is part of the permanent labour force, and although currently under-utilised, it is
policy of company to continue to pay Ghulaami labour in full. Acceptance of the contract would
reduce the idle time of Ghulaami labour.
Overheads:
The variable overheads incurred on each unit of Niks are at a constant rate of ` 40 per unit
including selling commission.
The fixed overhead absorption rate used by the company is ` 20 per labour hour of any type.
Additional administrative fixed cost to be incurred for Niks is ` 12 Lakhs.
Avoidable Fixed cost on diversion of labour force from other jobs for production of Niks is ` 2
Lakhs. This cost can be avoided only if Niks is manufactured.
Q. 41. B Ltd is a company that has in stock materials type XY that cost ` 75,000, but these are now obsolete
and have a scrap value of only ` 21,000. Other than selling the material for scrap, there are only two
alternative uses for them.
Alternative 1
Converting the obsolete materials into a specialized product which would require the following
additional work and material:
Material A
600 units
Material B
1,000 units
Direct Labour
5,000 hours unskilled
5,000 hours semi skilled
5,000 hours highly skilled
Extra selling & delivery expenses
` 27,000
Extra advertising
` 18,000

18

Ideal Classes Final CA Advanced Management Accounting Volume 1


The conversion would produce 900 units of saleable product and these could be sold for ` 300 per
unit.
Material A is already in stock and is widely used within the firm. Although present stocks together
with orders already planned will be sufficient to facilitate normal activity and extra material used by
adopting this alternative will necessitate such materials being replaced immediately.
Material B is also in stock but it is unlikely that any additional supplies can be obtained for some
considerable time, because of an industrial dispute. At the present time Material B is normally used
in the production of Product Z, which sells at ` 390 per unit and incurs total variable cost (excluding
Material B) of ` 210 per unit. Each unit of Product Z uses four units of Material B.
The details of Materials A and B are as follows:
Material A
` 100 per unit
` 85 per unit
` 90 per unit

Acquisition cost at the time of purchase


Net realizable value
Replacement cost

Material B
` 10 per unit
` 18 per unit

Alternative 2
Adopting the obsolete materials for use as a substitute for a sub assembly that is regularly used
within the firm details of the extra work and materials required are as follows:
Material C
Direct Labour :
4,000 hours unskilled
1,000 hours semi-skilled
4,000 hours highly skilled

1,000 units

1,200 units of the sub assembly are regularly used per quarter at a cost of ` 900 per unit. The
adaptation of material XY would reduce the quantity of the sub assembly purchased from outside
the firm to 900 units for the next quarter only. However since the volume purchased would be
reduced, some discount would be lost and the price of those purchased from outside would increase
to ` 1,050 per unit for that quarter.
Material C is not available externally though 1,000 units required would be available from stocks, it
would be produced as extra production. The standard cost per unit of Material C would be as
follows:
(`)
Direct materials
13
Direct labour
6 hours unskilled labour
18
Variable overhead
6 hours at ` 1
6
Fixed overhead
6 hours at ` 3
18
55
The wage rates and overhead recovery rates for B Ltd are:
Variable overhead
Fixed overhead
Unskilled labour
Semi skilled labour
Highly skilled labour

` 1 per direct labour hour


` 3 per direct labour hour
` 3 per direct labour hour
` 4 per direct labour hour
` 5 per direct labour hour

The unskilled labour is employed on a casual basis and sufficient labour can be acquired to exactly
meet the production requirements. Semi skilled labour is part of the permanent labour force, but
the company has temporary excess supply of this type of labour at the present time. Highly skilled
labour is in short supply and cannot be increased significantly in the short term; this labour is

19

Ideal Classes Final CA Advanced Management Accounting Volume 1


presently engaged in meeting the demand for product L, which requires 4 hours of highly skilled
labour. The contribution from the sale of one unit of product L is ` 24.
Given the above information you are required to present cost information advising whether the
stocks of Material XY should be sold converted into a specialized product (Alternative 1) or adopted
for use as a substitute for a sub assembly (Alternative 2).
(Nov 2000)
Q.42.

A company had nearly completed a job relating to construction of a specialized equipment, when it
discovered that the customer had gone out of business. At this stage, the position of the job was as
under:
(`)
Original cost estimate
1,75,200
Costs incurred so far
1,48,500
Costs to be incurred
29,700
Progress payment received from original customer
1,00,000
After searches, a new customer for the equipment has been found. He is interested to take the
equipment if certain modifications are carried out. The new customer wanted the equipment in its
original condition, but without its control device and with certain other modification. The costs of
these additions and modifications are estimated as under:
Direct materials (at cost)
Direct wages Dept : A
Dept : B
variable overheads
Delivery costs

` 1,050
15 man days
25 man days
25% of direct wages in each department
` 1,350

Fixed overheads will be absorbed at 50% of direct wages in each department.


The following additional information is available;
1.

The direct materials required for the modification are in stock and if not used for modification
of this order, they will be used in another job in place of materials that will now cost ` 2,250.

2.

Department A is working normally and hence any engagement of labour will have to be paid at
the direct wage rate of ` 120 per man day.

3.

Department B is extremely busy. Its direct wages rate is ` 100 per man day and it is currently
yielding a contribution of ` 3.20 per rupee of direct wages.

4.

Supervisory overtime payable for the modification is ` 1,050.

5.

The cost of the control device that the new customer does not require is ` 13,500.If it is taken
out, it can be used in another job in place of a different mechanism. The latter mechanism
has otherwise to be bought for ` 10,500.The dismantling and removal of the control
mechanism will take one man day in department A.

6.

If the proposal of new customer is not carried out, some of the materials in the original
equipment can be used in another contract in place of materials that would have cost ` 12,000.
It would have taken 2 man day of work in Dept A to make them suitable for this purpose. The
remaining materials will realize ` 11,400 as scrap. The drawings, which are included as part of
the job can be sold for ` 1,500
You are required to calculate the minimum price which the company can afford to quote for the new
customer as stated.
(May 2001)
Q.43.

ZED Ltd. operates two shops. Product A is manufactured in Shop 1 and customers jobs against
specific orders are being carried out in Shop 2.Its annual statement of income is:

20

Ideal Classes Final CA Advanced Management Accounting Volume 1


(`)
Particulars
Sales / Income
Material
Wages
Depreciation
Power
Rent
Heat & Light
Other Expenses
Total Costs
Net Income

Shop - 1
(Product - A)
1,25,000
40,000
45,000
18,000
2,000
5,000
500
4,500
1,15,000
10,000

Shop - 2
(Job Works)
2,50,000
50,000
1,00,000
31,500
3,500
30,000
3,000
2,000
2,20,000
30,000

Total
3,75,000
90,000
1,45,000
49,500
5,500
35,000
3,500
6,500
3,35,000
40,000

The depreciation charges are for machines used in the shops. The rent and heat and light are
apportioned between the shops on the basis of floor area occupied. All other costs are current
expenses identified with the output in a particular shop.
A valued customer has given a job to manufacture 5,000 units of X for Shop 2. As the company is
already working at full capacity, it will have to reduce the output of product A by 50% to accept the
said job. The customer is willing to pay ` 25 per unit of X. The material and labour will cost ` 10 and
` 18 respectively per unit. Power will be consumed on the job just equal to the power saved on
account of reduction of output of A. In addition the company has to incur additional overheads of `
10,000.
You are required to compute the following in respect of this job.
(a) Differential Cost
(b) Sunk Cost
(c) Full Cost
(d) Opportunity Cost
State whether the company should accept the job.
(May 1996)

Practice Section
Q.44. S Limited is engaged in manufacturing activities. It has received a request from one of its important
customers to supply a product which will require conversion of material M, which is a non-moving
item.
The following details are available:
Book value of material M
` 60
Realisable value of material M
` 80
Replacement cost of material M
` 100
It is estimated that conversion of one unit of M into one unit of the finished product will require
one labour hour. At present, labour is paid at the rate of ` 20 per hour. Other costs are as follows:
Out-of-pocket expenses
` 30 per unit
Allocated overheads
` 10 per unit
The labour will be re-deployed from other activities. It is estimated that the temporary
redeployment will not result in loss of contribution. The employees to be re-deployed are
permanent employees of the company.
Required:
Estimate the minimum price to be charged from the customer so that the company is not worse off
by executing the order.
(Nov 07, 4 Marks)

21

Ideal Classes Final CA Advanced Management Accounting Volume 1


Q.45. A research project, to date, has cost a company ` 2,50,000 and is under review. It is anticipated that,
should the project be allowed to proceed, it will be completed in about one year and can be sold for
` 4,00,000. The following additional information is available:
1. Materials have just been received for ` 60,000. These are extremely toxic, and if not used in the
project, have to be disposed of by special means at ` 15,000.
2. Labour: ` 75,000. The men are highly skilled. If they are released from the Research Project, they
may be transferred to the Works Department of the company and consequently the sales could
increase by ` 1,50,000. The accountant estimates that the prime cost of those sales would be `
1,00,000 and the overhead absorbed (all fixed) would amount to ` 25,000.
3. Research staff: ` 1,60,000. A decision has already been taken that this will be the last major
piece of research undertaken and consequently, when work on the project ceases, the staff
involved will be made redundant. Redundancy and severance pay have been estimated at `
25,000.
4. Share of General Building Expenses: ` 35,000.
The Managing Director is not sure what is included in this amount, but the accounts staff charge
similar amounts each year to each department.
You are required to advise whether the project should be allowed to proceed and explain the
reasons for the treatment of each of the amounts above in your analysis.
(May 07, 10 Marks)
Q. 46. Panchwati Cement Ltd. produces 43 grade cement for which the company has an assured market.
The output for 2004 has been budgeted at 1,80,000 units at 90% capacity utilisation. The cost sheet
based on output (per unit) is as follows:
`
Selling price
130
Direct material
30
Component EH
9.40
Direct wages @ ` 7 per hour
28
Factory overhead (50% fixed)
24
Selling and distribution overheads (75% variable)
16
Administrative overhead (fixed)
5
The factory overheads are applied on the basis of direct labour hours.
To utilise the idle capacity and to improve the profitability of the company, the following proposals
were put up before the Board of Directors for consideration:
(i) An order has been received from abroad for 500 units of product 53 grade cement per month
at ` 175 per unit. The cost data are:
Direct material ` 56 per unit, direct labour 10 hours per unit, selling and distribution overhead
applicable to this product order is ` 14 per unit and variable factory overhead are chargeable on
the basis of direct labour hours.
(ii) The company at present manufactures component EH, one unit of which is required for each
unit of product 43 grade. The cost details for 15,000 units of component EH are as follows:
`
30,000
52,500
25,500
33,000
1,41,000

Direct materials
Direct labour
Variable overheads
Fixed overheads
Total

The component EH however is available for purchase at the market at ` 7.90 per unit.

22

Ideal Classes Final CA Advanced Management Accounting Volume 1


(iii) In the event of company deciding to purchase the component EH from market, the company
has two alternatives for the use of the capacity so released, which are as under:
(a) Rent out the released capacity at ` 1 per hour.
(b) Manufacture component GYP which can be sold at ` 8 per unit. The cost data of this
component for 15,000 units are:
`
Direct materials
42,000
Direct labour
31,500
Factory Variable overheads
13,500
Other variable overheads
25,500
Total
1,12,500
Required:
1. Prepare a statement showing profitability of the company envisaged in the budget.
2. Evaluate the export order and state whether it is acceptable or not.
3. Make an appraisal of proposal to manufacture component EH and state whether the
component EH should be manufactured in the factory or purchased from the market. Assume
that no alternative use of spare capacity is available.
4. Evaluate the alternative use of the spare capacity and state whether to manufacture or buy the
component EH and if your decision is to buy the component EH, which of the two alternatives
for the use of spare capacity will you prefer?
(Nov 04, 16 marks)
Q. 47. Mr. X has taken a shop on lease and made a down payment of `2,50,000. Additionally, the rent
under lease amount is `96,000 per annum. If lease agreement is cancelled by Mr. X, then the initial
payment is forfeited. Mr. X plans to use the shop for the shop for the general stores business, and
has estimated operations for the next year as follows:
Sales
` 25,00,000
Less: value added tax (VAT)
` 2,80,000
Sales after VAT
22,20,000
Cost of goods sold
Wages and wages related cost
12,50,000
Rent including down payment
2,76,000
Rent including down payment
3,46,000
Rates, lighting and insurance
2,80,000
Audit, legal and general expenses
50,000
22,02,000
Net profit before tax
18,000
In the business, Mr. X will be devoting of half time, however no provision has been made for his
remuneration/salary. Mr. X also has an option to sublet the shop to his friend for a monthly rent of `
18,000, if he does not use the shop himself.
You are required to:
1. Identify the sunk and opportunity cost in the above problem.
2. State most profitable decision, which should be taken by Mr. X, supporting with appropriate
calculation.
(Nov 09, 4 Marks)

Home Work Section


Q. 48. X Ltd. has been approached by a customer who would like a special job to be done for him and is
willing to pay ` 22,000 for it. The job would require the following materials:

23

Ideal Classes Final CA Advanced Management Accounting Volume 1

Material
A
B
C
D

Total
units
Required
1,000
1,000
1,000
200

Book value of
units in stock
`/unit
2
3
4

Units
already in
stock
0
600
700
200

Realisable
value
`/unit
2.5
2.5
6

Replacement
cost
`/unit
6
5
4
9

1. Material B is used regularly by X Ltd. and if stocks are required for this job they would need to be
replaced to meet other production demand.
2. Materials C and D are in stocks as the result of previous excess purchase and they have a
restricted use. No other use could be found for material C but material D could be used in
another job as substitute for 300 units of material E which currently cost ` 5 per unit (of which
the company has no units in stock at the moment).
What are the relevant costs of materials, in deciding whether or not to accept the contract?
Assuming all other expenses on this contract to be specially incurred besides the relevant cost of
material is ` 550.
Q.49.

Tiptop Textiles manufactures a wide range of fashion fabrics. The company is considering whether to
add a further product the Superb to the range. A market research survey recently undertaken at a
cost of ` 50,000 suggests that demand for the Superb will last only for one year, during which
50,000 units could be sold at ` 18 per unit. Production and sale of Superb would take place evenly
throughout the years. The following information is available regarding the cost of manufacturing
Superb.
Raw Material:
Each Superb would require 3 types of raw material Posh, Flash and splash. Quantities required,
current stock levels and cost of each raw material are shown below. Posh is used regularly by the
company and stocks are replaced as they are used. The current stock of Flash is the result of
overbuying for an earlier contract. The material is not used regularly by Tiptop Textiles and any stock
that was not used to manufacture Superb would be sold. The company does not carry a stock of
Splash and the units required would be specially purchased.

Raw
Material

Posh
Flash
Splash

Quantity
required
Per unit
of Superb
(metres)
1
2
0.5

Current
Stock
Level

Costs per metre of raw material


Original
Current
Current
cost
replaceresale
ment cost
value

(metres)
1,00,000
60,000
0

`
2.1
3.3

`
2.5
2.8
5.5

`
1.8
1.1
5.5

Labour:
Production of each Superb would require a quarter of an hour of skilled labour and two hours of
unskilled labour. Current wage rates are ` 3 per hour for skilled labour and ` 2 per hour for unskilled
labour.
In addition, one foreman would be required to devote all his working time for one year in
supervision of the production of Superb. He is currently paid an annual salary of ` 15,000.
Tiptop Textiles is currently finding it very difficult to get skilled labour. The skilled workers needed to
manufacture Superb would be transferred from another job on which they are earning a
contribution surplus of ` 1.50 per labour hour, comprising sales revenue of ` 10.00 less skilled labour

24

Ideal Classes Final CA Advanced Management Accounting Volume 1


wages of ` 3.00 and other variable costs of ` 5.50. It would not be possible to employ additional
skilled labour during the coming year.
Because the company intends to expand in the future, it has decided not to terminate the services of
any unskilled worker in the foreseeable future who are idle at present.
The foreman is due to retire immediately on an annual pension of ` 6,000 payable by the company.
He has been prevailed upon to stay on for a further year and to defer his pension for one year in
return for his annual salary.
Machinery:
Two machines would be required to manufacture Superb MT 4 and MT 7. Details of each machine
are as under:
Start of the
End of the
year
year
`
`
MT 4
Replacement cost
80,000
65,000
Resale value
60,000
47,000
MT 7
Replacement cost
13,000
9,000
Resale value
11,000
8,000
Straight line depreciation has been charged on each machine for each year of its life. Tiptop Textiles
owns a number of MT 4 machines, which are used regularly on various products. Each MT 4 is
replaced as soon as it reaches the end of its useful life. MT 7 machines are no longer used and the
one which would be used for Superb is the only one the company now has. If it was not used to
produce Superb, it would be sold immediately.
Overheads: A predetermined rate of recovery for overhead is in operation and the fixed overheads
are recovered fully from the regular production at ` 3.50 per labour hour. Variable overhead cost for
Superb is estimated at ` 1.20 per unit produced.
For the decision making, incremental cost based on relevant cost and opportunity cost are usually
computed.
You are required to compute such a cost sheet for Superb with all details of materials, labour,
overheads etc., substantiating the figures with necessary explanation.
Q. 50. Mahila Griha Udyog Industries is considering to supply its products a special range of namkeens to
a departmental store. The contract will last for 50 weeks, and the details are given below:
`
1,50,000
1,80,000
3,00,000

Material:
X (in stock at original cost)
Y (on order on contract)
Z (to be ordered)
Labour:
Skilled
Non-skilled
Supervisory
General overheads
Total cost
Price offered by departmental store
Net Loss

5,40,000
3,00,000
1,00,000
10,80,000
26,50,000
18,00,000
8,50,000

Should the contract be accepted if the following additional information is considered?


1. Material X is an obsolete material. It can only be used on another product, the material for
which is available at ` 1,35,000 (Material X requires some adaptation to be used and costs `
27,000)

25

Ideal Classes Final CA Advanced Management Accounting Volume 1


2. Material Y is ordered for some other product which is no longer required. It now has a residual
value of ` 2,10,000.
3. Skilled labour can work on other contracts which are presently operated by semi-skilled labour
at a cost of ` 5,70,000.
4. Non-skilled labour are specially employed for this contract.
5. Supervisory staff will remain whether or not the contract is accepted. Only two of them can
replace other positions where the salary is ` 35,000.
6. Overheads are charged at 200% of skilled labour, Only ` 1,25,000 would be avoidable, if the
contract is not accepted.
(May 1999)
Q. 51. Star Bicycle Company, produced and sold 1,10,000 bicycles annually, under the brand name Smart
with a price tag ` 899 per bicycle. Like all other players in the industry, Star too was running under
capacity. The manufacturing cost of these cycles for each unit was-material ` 300, labour ` 200 and
Manufacturing overhead ` 300, 40% of the manufacturing overhead was variable. General and
administration expenses were 50% of labour cost.
Star has now received a proposal to sell 25,000 bicycles per year under the brand name Jeet to a
chain store at a price of ` 800 per bicycle. The brand will be exclusive for the chain stores as they will
market it as their own product. Expenditure for producing Jeet will be the same as that of Smart, as
design of Jeet will exactly be same as that of Smart with only some cosmetic changes. To produce
Jeet however, ` 6,00,000 additional funds will be required on an average. Further it estimated that
sale of Jeet through the store will reduce the sale of Smart by 10,000 units.
You are required to calculate the relevant cost of Jeet, given that the weighted average cost of
capital of Star Co. is 15%. Also show the impact of this proposal on companys profit.
(ICWA Dec 2002)
Q.52.

A Ltd. produces and markets a range of consumer durable appliance. It ensures after sales service
through X Ltd. The big appliances are serviced at customers residence while small appliances are
serviced at workshop of X Ltd.
The material supplied to X Ltd. is charged at cost plus 10%. X Ltd charges customers at 25% over the
above price.
For labour, the company receives 10% of the rate fixed for work done under the after sales service
agreement and 15% of the rate fixed in case of jobs not covered under the agreement, from X Ltd.
60% by value of the total work undertaken by X ltd. was for big appliances and rest accounted for
small appliances during the previous year.
The company decided to carry out all or some of the work itself and has chosen one area in the first
instance. During the previous year the company earned a profit of ` 2,16,000 as detailed below from
X Ltd. for the chosen area:
Material
`
60,000
20,000

Under after sales service agreement


For jobs not covered under the agreement

Labour
`
1,00,000
36,000

The company forecasts same volume of work in that area for the ensuing period. The following three
options are under consideration of the management:
1.
To set up a local service centre to provide service for small appliances only. The existing
system is to continue for big appliances.
2.
To set up a local service centre to provide service for big appliances only. The existing system
is to continue for small appliances.
3.
To set up a local service centre to provide service for all appliances. The existing system then
stands withdrawn.

26

Ideal Classes Final CA Advanced Management Accounting Volume 1


The relevant costs for carrying out jobs under the above options are as under:

Heat, rent, light etc.


Management cost
Service staff costs
Transport costs

Option 1
125
108
230
25

Option 2
50
83
440
220

(` ' 000)
Option 3
150
150
750
230

You are required to find out the most profitable option.


(Nov 1996)
Q.53.

Forward Foundry Ltd. is feeling the effects of a general recession in the industry. Its budget for the
coming half year is on an output of only 500 tonnes of casting a month, which is less than half of its
capacity. The price of casting varies with the composition of the metal and the shape of the mould,
but they average ` 175 a tonne. The following details are from the Monthly production Cost Budget
at 500 tonne level.

Labour
Variable Overhead
Fixed overhead
Total
Labour and overhead per direct hour

Core
making
`
10,000
3,000
5,000
18,000

Melting and
Pouring
`
16,000
1,000
9,000
26,000

Moulding
`
6,000
1,000
2,000
9,000

Cleaning and
grinding
`
4,500
1,000
1,000
6,500

6.5

5.2

Operation at this level has brought the company to the brink of break-even. It is feared that if the
lack of work continues, the company may have to lay off some of the most highly skilled workers
whom it would be difficult to get back when the volume picks up later on. No wonder, the Works
Manager at this juncture welcome an order for 90,000 castings, each weighing about 40 lbs., to be
delivered on a regular schedule during the next six months. As the immediate concern of the Work
Manager is to keep his work force occupied, he does not want to lose the order and is ready to
recommend a quotation on a no-profit and no-loss basis.
Materials required would cost ` 1 per casting after deducting scrap credits. The direct labour hours
per casting required for each department would be:
0.09
Core making
Melting and pouring
0.15
Moulding
0.06
Cleaning and grinding
0.06
Variable overhead would bear a normal relationship to labour cost in the melting and pouring
department and in the moulding department. In core making, cleaning and grinding, however, the
extra labour requirements would not be accompanied by proportionate increases in variable
overhead. The total variable overhead would increase by ` 1.20 for every additional labour hour in
core making and by 30 paise for every additional labour hour in cleaning and grinding. Standard
wage rates are in operation in each department and no labour variances are anticipated.
To handle an order as large as this, certain increases in factory overheads would be necessary
amounting to ` 1,000 a month for all departments put together. Production for this order would be
spread evenly over the six months period.
You are required to:
1. Prepare a revised monthly Labour and Overhead Cost Budget, reflecting the
order.

27

addition of this

Ideal Classes Final CA Advanced Management Accounting Volume 1


2. Determine the lowest price at which quotation can be given for 90,000 castings without
incurring a loss.

Q. 54. A company manufactures two products AB and CD by utilizing 25% and 40% of its total capacity
respectively. The cost data per unit for 2003-04 are as under:
Particulars
Production & sales
Selling price
Direct material
Direct labour (` 5 per hour)

AB'
5,000
80
10
25

(units)
(`)
(`)
(`)

CD'
10,000
100
30
20

Variable overheads are 100% on wages. Fixed overheads for 2003-04 amounted to ` 2,25,000.
During 2004-05, the company expects that the direct material costs will rise by 5% the labour hourly
rate will rise by 25 paise and variable overheads will continue to maintain same relationship with
wages as was in 2003-04. For the same volume of output as was in 2003-04, the selling price is to be
enhanced by 5% in case of AB and 4% in case of CD.
The company has the following proposals for consideration of the management for 2004-05 to
improve profitability:
(a) Utilize the balance capacity to produce AB and to sell this increased production at the
existing selling price of ` 80.
(b) Utilize the balance capacity to produce CD, while doing so the efficiency will however go
down by 16% on account of newly recruited labour in respect of this increased production.
Fixed selling and distribution expenses of ` 50,000 will have to be spent to sell this additional
output.
(c) Introduce new product EF to utilize the balance capacity, one unit of EF can be
manufactured in 7 labour hours. Direct material will cost ` 40 per unit. Its selling price per unit
will be ` 145. Variable overheads will maintain same ratio to wages as for other two products.
To boost the sales of EF special advertising expenses of ` 30,000 will be spent.
The present allocation of 25% and 40% capacities for AB and CD cannot be changed and only the
spare capacity is required to be used for production under the aforesaid proposals.
Required:
1. Present a statement of profit for 2003-04
2. Using incremental revenue and differential cost approach, find out which proposal is more
profitable for 2004-05.
3. Present a statement of profit for 2004-05 based on above recommendation.
(Nov-1995)

28

Ideal Classes Final CA Advanced Management Accounting Volume 1


Key Factor or Limiting Factors Analysis
Marginal costing can also be used in budgeting, to help management to determine the profit maximising
budget. Planning is necessary when one or more factors of production or other business resources are short
in supply. Marginal costing really shows its merit when scarce resources are being considered. Examples of
resource restrictions which may apply are as follows:

Limit to the availability of a particular grade of labour

Shortage of raw material

Limit of machine capacity

Shortage of cash to finance production (working capital)

If labour supply, materials availability, machine capacity or cash availability limit production to less than the
volume which could be achieved, management is faced with the problem of deciding what to produce and
what not to produce, because there are insufficient resources to make everything. The limiting factor is
often sales demand itself, in which case the business should produce enough goods or services to meet the
demand in full, provided that sales of the goods earn a positive contribution towards fixed costs and profits.
However, when the limiting factor is a production resource, the business must decide which part of sales
demand it should meet, and which part must be left unsatisfied. Marginal costing analysis can be used to
indicate the profit-maximising.
Analysis when only one Limiting Factor exists:
To maximise the profit, the contribution must be maximised, within the constraints given. The simple
approach to tackle this kind of situation would be as follows:
Step 1 Identify the Limiting Factor or Key Factor.
Doubt: How to identify whether a key factor exist?

i.
ii.
iii.

That is very simple:


Calculate the volume of resources required to produce maximum units demanded
Calculate the volume of resources available
Compare the two volumes above; if (i) exceed (ii), there is a limiting factor.

Step 2

Ascertain the contribution per unit of the various products.

Step 3

Ascertain the contribution per unit of the Key Factor. For example, if raw material is the Key Factor,
calculate the contribution per kg of the raw material; if labour is in short supply, calculate the
contribution per hour of the labour, etc.

Step 4

Rank the products on the basis of the contribution per unit of the Key Factor. (The product with the
highest contribution per unit of the scarce resource should receive priority in the allocation of the
resource in the production budget)

Step 5

Allocate the scarce resources on the basis of the ranks given above, taking into account the
maximum demand.

29

Ideal Classes Final CA Advanced Management Accounting Volume 1

Section 3 Key Factor


Q.55.

(Key Factor Simple)


X Ltd manufactures and sells 3 products, details of which are mentioned below:
No. of units produced and sold
Selling Price per unit (`)
Direct Material Cost per unit (`)
(@` 3/- per Kg)
Direct Labour Cost per unit (`)
(@` 4/- per Hr.)
Other Variable Cost per unit (`)

A
20,000
50
15

B
20,000
50
9

C
20,000
50
6

20

28

32

The total fixed cost for the year is ` 2,40,000.


You are required to:
1.
Prepare a statement of operating income for the current year.
2.
If in the next year there is shortage of Direct Materials, which will be available only to the
extent of 80% of the total requirement, find the optimal product mix for the next year in order
to maximize the profit. Assume that the number of units sold at present is the maximum
possible sale.
Q.56.

(Key Factor Desirable Capacity Level)


The relevant data of X Ltd. for its three products A, B and C are as under:
Particulars
Direct Material
Direct Labour
Variable Overheads
Selling Price
Machine Hours Required

A
260
130
110
860
12

B
300
270
230
1040
6

C
250
260
180
930
3

The estimated fixed overheads at three different levels of 6,000, 8,400 and 10,800 machine hours
are ` 1,50,000, ` 2,20,000 and ` 3,00,000 respectively. The maximum demand of A, B and C in a cost
period are 500, 300 and 1,800 units respectively.
You are required to find out
1.
The most profitable product-mix at each level and
2.
The level of activity where the profit would be maximum.
(May 1997)
Q.57.

(Key Factor Make or Buy)


Fortune Ltd. manufactures Product N using one unit each of three components named P, Q & R and
sells it at ` 37.50 per unit. It has two divisions. In production division it produces all the types of
components by using its full capacity of 42,000 machine hours. In assembly division the remaining
job is performed by the workers manually before N is ready for sale:
Product N is manufactured in batches of 100 units and the data relating to the current production
per batch are:

30

Ideal Classes Final CA Advanced Management Accounting Volume 1


Particulars

Production Division
Component - P
Component - Q
Component -R
Assembly Division
Assembly

Machine
hours

Variable
costs
`

Fixed
Costs
`

Total
costs
`

15
25
30

375
450
450

150
175
450

525
625
900

800
2,075

325
1,100

1125
3,175

For the next year the company has estimated that its sale would go up by 50% more than the
present sales and probably even by 75% if the production capacity is made available.
The machine capacity cannot be increased during the next year even though the workers in the
assembly division can be increased as per requirement without any increase in fixed costs. To meet
the increased demand, production can be taken up and processed in assembly division by procuring
the components from the open market. The company has received the following price quotations for
the purchase of components:
P
5.55

Price offered per component

Q
7.00

(`)
R
8.40

You are required to:


1. Determine the production and profits being earned at present.
2. Indicate which of the component(s) should be purchased and in what quantities at the two
estimated levels of output viz. increase by 50% and 75% of existing production.
3. Prepare a statement showing the companys profitability at both the estimated levels of output.
(May 1994)
Q. 58. (Key Factor Make or Buy)
Priya Gadgets Ltd. specializing in household gadgets, has just perfected and test marketed a
modified version of a popular gadgets. It has three components X, Y and Z one of each is required
per gadget. All these components are made and assembled in its own factory and capacity utilization
of the machines is full.
The modification essentially involves a special machining and fixing a new attachment for which the
company has provided for double the existing production capacity to take care of possible increased
demand.
The cost structure of the modified gadget is as under:
Component

X
Y
Z
Special machining & assembly

Machine
hours
per unit
Hours
16
24
32
-

Selling price

Variable
cost
per unit
`
50
56
54
60
220

Fixed cost
allocated
per unit
`
15
20
30
45
110

Total
cost
`
65
76
84
105
330
500

Since the response to the modified gadgets is very good the Company would like to capture the
market in the ensuing year itself by increasing sales. While all the existing machines in the factory
are capable of making all the components X, Y and Z, increase of machine capacity can not be

31

Ideal Classes Final CA Advanced Management Accounting Volume 1


achieved / made during the budget year. However the special machining process and capacity
permits one of the components, either X, Y or Z to be bought from outside. The following offers have
been received:
Component
X
Y
Z

Price per unit


` 66
` 78
` 94

The marketing manager feels that sales can be increased at least by 50% during the year and with a
little advertisement support even 75%.
You are required to give your recommendation as to which components should be bought from
outside if production is to be increased by 50% and 75% respectively.
(I.C.W.A. June, 1996)
Q. 59. (Key Factor Make or Buy)
K Ltd. manufactures and sells a range of sport goods. Management is considering a proposal for an
advertising campaign which would cost the company ` 3,00,000. The marketing department has put
forward the following two alternative sales budgets for the following year:
(000 units)
Products
Particulars
A
B
C
D
Budget 1 -Without Advertising
216
336
312
180
Budget 2 -With Advertising
240
372
342
198
Selling prices and variable production costs are budgeted as follows:
(` Per unit)
Products
Particulars
Selling Prices
Variable Production Costs:
- Direct Material
- Direct Labour
- Variable Overheads

A
11.94

B
14.34

C
27.54

D
23.94

5.04
2.04
0.72

6.60
2.04
0.72

15.24
3.36
1.20

12.48
3.18
1.08

Other Data:
1.
The variable overheads are absorbed on a machine hour basis at a rate of ` 1.20 per machine
hour.
2.
Fixed overheads total ` 30,84,000 per annum.
3.
Production capacity during the budgeted period is 8,15,000 machine hours.
4.
Products A and C could be bought in at ` 10.68 per unit and ` 24 per unit respectively.
Required:
(i)
Determine whether investment in the advertising campaign would be worthwhile and how
production facilities would be best utilized.
(ii) Explain the assumptions and reasoning behind your advice.
(May 1996)
Q.60.

An agriculturist has 480 hectares of land on which he grows potatoes, tomatoes, peas and carrots.
Out of the total area of land, 340 hectares are suitable for all the four vegetables but the remaining
140 hectares of land are suitable only for growing peas and carrots. Labour for all kinds of farm work
is available in plenty.

32

Ideal Classes Final CA Advanced Management Accounting Volume 1


The market requirement is that all the four vegetables must be produced with a minimum of 5,000
boxes of any one variety. The farmer has decided that the area devoted to any crop should be in
terms of complete hectares and not in fractions of a hectare. The only other limitation is that not
more than 1,13,750 boxes of any one vegetable should be produced.
The relevant data concerning production, market prices and costs are as under:
Annual yield:
Boxes per hectare
Costs:
Direct material per hectare
Direct labour:
Growing per hectare
Harvesting & packing per box
Transport per box
Market price per box

Potatoes

Peas

Carrots

Tomatoes

350

100

70

180

` 952

` 432

` 384

` 1,792
` 7.20
` 10.40
` 30.76

` 1,216
` 6.56
` 10.40
` 31.74

` 744
` 8.80
` 8.00
` 36.80

` 624
` 1,056
` 10.40
` 19.20
` 44.55

Fixed expenses per annum:


Growing
` 1,24,000
Harvesting
` 75,000
Transport
` 75,000
General administration
` 1,50,000
It is possible to make the land presently suitable for Peas and Carrots, viable for growing Potatoes
and Tomatoes as well, if certain land development work is undertaken. This work will involve a
capital expenditure of ` 6,000 per hectare, which a bank is prepared to finance @ 15% p.a. If such
improvement is undertaken, the harvesting cost of the entire crop of Tomatoes will decrease on an
average by ` 2.60 per box.
Required:
1. Calculate, within the given constraints, the area to be cultivated in respect of each crop to
achieve the largest total profit and the amount of such total profit before land development
work is undertaken.
2. Assuming that the other constraints continues, advice the grower whether the land
development scheme should be undertaken and if so the maximum total profit that would be
achieved after the said land development scheme is undertaken.
Q.61.

(Key Factor Pricing Decision)


Universe Ltd. manufactures two products X and Y. it is facing severe competition in the market. The
monthly sales potential in units at different selling prices as anticipated by the sales manager are as
under:
Product X
Product Y
Selling Price
Sales Potential
Selling Price
Sales Potential
Per Unit (`)
(in units)
Per Unit (`)
(in units)
110
5,000
78
30,000
108
7,500
77
32,000
107
8,000
75
35,000
103
8,400
72
40,000
96
9,000
69
45,000
The total costs as disclosed by the budgets of the company are as follows:

33

Ideal Classes Final CA Advanced Management Accounting Volume 1


Product X
Output and sales per month (units)
Total costs per month (` In lacs)
Labour hours needed per month

Product Y

5,000
5

9,000
6.6

20,000

36,000

30,000
18

45,000
25.5

60,000

90,000

You are required to find out the selling price and units to be sold to earn maximum profit where:
1.
Labour hours are available without any restrictions and
2.
Only 95,000 labour hours are available.
Q.62.

(Key Factor Margin of Safety)


On a turnover of ` 20 crores in 1984, a large manufacturing company earned a profit of 10% before
interest and depreciation which were-fixed. The product mix was as under:
Products
P
Q
R
S

Mix %
to total sales
10
30
20
40

PV ratio
%
30
20
40
10

Raw material
as % on sales value
40%
35%
50%
60%

Interest and depreciation amounted to ` 150 lacs and ` 77 lacs respectively.


Due to fluctuations in prices in the International Market, the company anticipates that the cost of
raw materials which are imported will increase by 10% during 1985. The company has been able to
secure a license for the import of raw materials of a value of ` 1,023 lacs at 1985 prices. In order to
counteract the increase in costs of raw materials the company is contemplating to revise its product
mix. The market survey report recently prepared indicates that the sales potential of each of the
products P, Q and R can be increased up to 30% of the total sales value of 1984. There is no
inventory of finished goods or work in process in both the years.
1. Set an optimal product mix for 1985 and find the profitability.
2. What percentage increase in overall price is required in 1985 to raise the sales value to maintain
the margin of safety at 10%.

Practice Section
Q. 63. Zilmil Ltd. makes two products Brightly; and Lightly. Both the products use the same labour force,
the size of which is restricted to 78,000 hours per month. Brightly needs 2 hours per unit to make
whereas lightly needs one hour. The estimated production and sales, manufacturing and selling
expenses per month are as follows:
Brightly
12,000
16,000
34,00,000
38,00,000

Production and Sales (in Nos.)


Cost per month (`)

Lightly
40,000
48,000
62,00,000
66,80,000

The Company is considering pricing option in a highly competitive market. It has estimated sales
demand at various selling prices:
Brightly:
Selling Price per unit (`)
Sales demand per month

276
12,000

34

272
14,000

262
16,000

264
16,000

260
20,000

254
22,000

Ideal Classes Final CA Advanced Management Accounting Volume 1


Lightly:
Selling Price per unit (`)
Sales demand per month

163
40,000

162
42,000

161
44,000

160
46,000

156
48,000

152
50,000

You are required to compute profit maximizing price and quantity for each product.
(May 06, 11 Marks)
Q. 64. E Ltd. manufactures and sells four types of products under the brand names A, B, C and D. On a
turnover of ` 30 crores in 2009, company earned a profit of 10% before interest and depreciation
which are fixed. The details of product mix and other information are as follows:
Products
Mix % to total sales
PV Ratio (%)
Raw Material as % on
sales value
A
30
20
35
B
10
30
40
C
20
40
50
D
40
10
60
Interest and depreciation amounted to ` 225 lakhs and ` 115.50 lakhs respectively. Due to increase
in prices in the international market, the company anticipates that the cost of raw materials which
are imported will increase by 10% during 2010. The company has been able to secure a licence for
the import of raw materials of a value of ` 1,535 lakhs at 2010 prices. In order to counteract the
increase in costs of raw materials, the company is contemplating to revise its product mix. The
market survey report indicates that the sales potential of each of the products: A, B and C can be
increased upto 30% of total sales value of 2009. There was no inventory of finished goods or work in
progress in both the years.
You are required to:
Set an optional product mix for 2010 and find the profitability.
(Nov 2010, 12 Marks)
Q. 65. A manufacturer produces three products whose cost data are as follows:
Particulars
X
Y
32.00
76.00
Direct materials (`/unit)
Direct Labour:
Hours
Hours
Deptt.
Rate / hour (`)
1
2.50
18
10
2
3.00
5
4
3
2.00
10
5
8
4.50
Variable overheads (`)

Z
58.50
Hours
20
7
20
10.50

Fixed overheads (`) 4,00,000 per annum.


The budget was prepared at a time, when market was sluggish. The budgeted quantities and selling
prices are as under:
Product
Budgeted quantity (Units)
Selling Price / unit (`)
X
19,500
135
Y
15,600
140
Z
15,600
200
Later, the market improved and the sales quantities could be increased by 20 per cent for product X
and 25 per cent each for product Y and Z. The sales manager confirmed that the increased sales
could be achieved at the prices originally budgeted. The production manager stated that the output
could not be increased beyond the budgeted level due to the limitation of Direct labour hours in
department 2.

35

Ideal Classes Final CA Advanced Management Accounting Volume 1


Required:
1. Prepare a statement of budgeted profitability.
2. Set optimal product mix and calculate the optimal profit.
(Nov 07, 14 Marks)
Q.66. R Ltd. has spare capacity in two of its manufacturing departments Department 4 and
Department 5. A five-day week of 40 hours is worked, but there is only enough internal work for
3 days per week so that 2 days per week (16 hours) could be available in each department. R Ltd.
has sold this time to another manufacturer, but there is some concern about the profitability of
this work.
The accountant has prepared a table giving the hourly operating cost in each department. The
summarised figures are as follows:
Department 4
Power costs
Labour costs
Overhead costs

Department 5

40
40
40
120

60
20
40
120

The labour is paid on a time basis and there is no charge in the weekly wage bill whether or not
the plant is working at full capacity. The overhead figures are based on firms current overhead
absorption rates (fixed and variable) when the departments are operating at 90% of full capacity
(assume a 50 week year). The budgeted fixed overhead attributed to department 4 is ` 36,000
p.a. and that for Department 5 ` 50,400 p.a.
As a short term measure the company has been selling processing time to another manufacturer
@ ` 70 per hour in either departments. The customer is willing to continue this arrangement and
to purchase any spare time available, but R Ltd. is considering the introduction of a new product
on a minor scale to absorb the spare capacity.
Each unit of the new product would require 45 minutes in Department 4 and 20 minutes in
Department 5. The variable cost of the required input material is ` 10 per unit. The market
study indicated as follows:
1. with a selling price of ` 100, the demand would be 1,500 units p.a.
2. with a selling price of ` 110, the demand would be 1,000 units p.a.
3. with a selling price of ` 120, the demand would be 500 units p.a.
You are required to calculate the best weekly programme for the spare time in the two
manufacturing departments, to determine the best price to charge for the new product and to
quantity the weekly gain that this programme and price should yield.
(May 05, 11 Marks)
Q. 67. Bloom Ltd. makes 3 products, A, B and C. The following information is available:
(Figures in ` per unit)
A
B
C
Selling price (peak-season)
550
630
690
Selling price (off-season)
550
604
690
Material cost
230
260
290
Labour (peak-season)
110
120
150
Labour (off-season)
100
99
149
Variable production overhead
100
120
130
Variable selling overhead(only for peak-season)
10
20
15
Labour hours required for one unit of production
8
11
7 (hours)

36

Ideal Classes Final CA Advanced Management Accounting Volume 1


Material cost and variable production overheads are the same for the peak-season and off-season.
Variable selling overheads are not incurred in the off-season. Fixed costs amount to ` 26,780 for
each season, of which ` 2,000 is towards salary for special technician, incurred only for product B,
and ` 4,780 is the amount that will be incurred on after-sales warranty and free maintenance of only
product C, to match competition.
Labour force can be interchangeably used for all the products. During peak-season, there is labour
shortage and the maximum labour hours available are 1,617 hours. During off-season, labour is
freely available, but demand is limited to 100 units of A, 115 units of B and 135 units of C, with
production facility being limited to 215 units for A, B and C put together.
You are required to:
1. Advise the company about the best product mix during peak-season for maximum profit.
2. What will be the maximum profit for the off-season?
(Nov 08, new, 8 Marks)
Q.68.

An agro-products producer company is


information is relating to the current year:
Products/Corps
Area occupied (acres)
Yield per acre (ton)
Selling price per ton (`)
Variable cost per acre (`)
Seeds
Pesticides
Fertilizers
Cultivations
Direct wages

planning its production for next year. The following


A1
250
50
200

A2
200
40
250

B1
300
45
300

B2
250
60
270

300
150
125
125
4,000

250
200
75
75
4,500

450
300
100
100
5,000

400
250
125
125
5,700

Fixed overhead per annum (`) 53,76,000.


The land that is being used for the production of B1 and B2 can be used for either crop, but not for
A1 and A2. The land that is being used for A1 and A2 can be used for either crop, but not for B1 and
B2. In order to provide adequate market service, the company must produce each year t least 2,000
tons each of A1 and A2 and 1,800 tons each of B1 and B2.
You are required to:
1. Prepare a statement of the profit for the current year.
2. Profit for the production mix by fulfilling market commitment.
3. Assuming that the land could be cultivated to produce any of the four products and there was no
market commitment, calculate: Profit amount of most profitable crop and breakeven point of
most profitable crop in terms of acres and sales value.
(Nov 09, 11 Marks)
Q.69.

Lee Electronic manufactures four types of electronic products, A, B, C and D. All these products have
a good demand in the market. The following figures are given to you:
A
B
C
D
64
72
45
56
Material cost (`/u)
48
32
64
24
Machining Cost (`/u @ ` 8 per hour)
32
36
44
20
Other variable costs (`/u)
162
156
173
118
Selling Price (`/u)
Market Demand (Units)
52,000
48,500
26,500
30,000
Fixed overhead at different levels of operation are:

37

Ideal Classes Final CA Advanced Management Accounting Volume 1


Total fixed cost (`)
10,00,000
10,50,000
11,50,000

Level of operation (in production hours)


Upto 1,50,000
1,50,000 30,00,000
4,50,000- 6,00,000

At present, the available production capacity in the company is 4,98,000 machine hours. This
capacity is not enough to meet the entire market demand and hence the production manager wants
to increase the capacity. The company wants to retain the customers by meeting their demands
through alternative ways. One alternative is to sub-contract a part of its production. The subcontract offer received as under:
A
B
C
D
146
126
155
108
Sub-contract Price (`/u)
The company seeks your advice in terms of products and quantities to be produced and/or subcontracted, so as to achieve the maximum possible profit. You are required to also compute the
profit expected from your suggestion.
(Nov 09, 18 Marks)
Q. 70. X Ltd. supplies spare parts to an air craft company Y Ltd. The production capacity of X Ltd. facilitates
production of any one spare part for a particular period of time. The following are the cost and other
information for the production of the two different spare parts A and B:
Per Unit
Part A
Part B
Alloy usage
1.6 kgs
1.6 kgs
Machine Time: Machine A
0.6 hr
0.25 hr
Machine Time: Machine B
0.5 hr
0.55 hr
Target Price (`)
145
115
Total hours available: Machine A
4,000 hours
Machine A
4,500 hours
Alloy available is 13,000 kgs. @ ` 12.50 per kg.
Variable overheads per machine hours:
Machine A: ` 80
Machine B: ` 100
You are required to identify the spare part which will optimize contribution at the offered price.
If Y Ltd. reduces target price by 10% and offers ` 60 per hour of unutilised machine hour, what will
be the total contribution from the spare part identified above?
(May 2010, 8 Marks)

Home Work Section


Q. 71. (Key Factor Desirable Capacity Level)
Something More Ltd. is considering adding products to its product line. After a lot of deliberations
between the sales and production personnel, it is decided that products P, Q and R would be the
most desirable additions to the companys product range on account of the technical competency,
marketing potentials and production flexibility as regards these products. In fact, P, Q and R can all
be made on the same kind of plant as that is already in use and therefore as regards production, all
products can be readily interchanged. However, it is considered necessary to build further plant
facilities to cater for this additional production. In this connection the following data are relevant:

38

Ideal Classes Final CA Advanced Management Accounting Volume 1


Products
Direct material per unit
Direct labour per unit
Variable overheads per unit
Selling price per unit
Demand in units per cost period
(on the basis of the above selling price
Machine hours required per unit of production

P
`
100
50
50
350
200

Q
`
120
70
130
420
125

R
`
90
90
100
370
750

15

It is felt that initially extra plant facilities can be built to operate at the following five different levels
of activity viz. 1,800, 2,300, 2,800, 3,300 and 3,800 machine hours per cost period. The fixed
overhead costs for a cost period relevant to these five different levels of activity are estimated at `
15,000 ` 20,000, ` 26,000, ` 33,0000 and ` 39,000 respectively.
You are required to advice, with supporting figures, the product or products to be manufactured and
in what quantities at each of the five contemplated levels of activity that would seem most desirable
to be pursued for maximization of profits.
Q.72.

(Key Factor Overtime Possibility)


A firm produces 5 different products from a single raw material. Raw material is available in
abundance at ` 6 per kg. The labour rate is ` 8 per hour for all products. The plant capacity is 21,000
labour hours for the budget period. Production facilities can produce all the products. The factory
overhead rate is ` 8 per hour, comprising ` 5.60 per hour as fixed overhead and ` 2.40 per hour as
variable overhead. The Selling Commission is 10 per cent of the product price. Given the following
information, you are to suggest a suitable sales mix which will maximize the Companys profits.
Determine the profits that will be earned at the selected sales mix.
Product

A
B
C
D
E

Q.73.

Market
demands
(units)
4,000
3,600
4,500
6,000
5,000

Selling
price
(`)
32.00
30.00
48.00
36.00
44.00

Labour Hours
required per
Unit
1.00
0.80
1.50
1.10
1.40

Raw Material
required per
unit (in gms.)
700
500
1,500
1,300
1,500

Assume, in above situation, 3,500 hours of overtime working is possible. It will result in additional
fixed overheads of ` 20,000, a doubling of labour rates and a 50% increase in variable overhead rates
for such overtime. Do you recommend the overtime working?
(May 1992)
(Key Factor Make or Buy)
Agrocaps Ltd., engaged in manufacturing agriculture machinery, is preparing its annual budget for
the coming year. The company has a metal pressing capacity of 20,000 hours, which will be
insufficient for manufacture of all requirements of components A, B, C and D.
The company has the following choice to overcome the shortage of capacity:1. Buy the components from outside suppliers.
2. Buy from outside suppliers and/or use a partial second shift.
Standard Production cost per unit

39

Ideal Classes Final CA Advanced Management Accounting Volume 1


Component
Variable Cost :
Direct materials
Direct wages
Direct expenses
Fixed overhead
Total production cost
Requirement in units

(`)
D

37
10
10
5
62
2000

27
8
20
4
59
3500

25
22
10
11
68
1500

44
40
60
20
164
2800

Direct expenses relate to the use of the metal presses which cost ` 10 per hour, to operate. Fixed
overheads are absorbed as a percentage of direct wages.
Supply of all or any part of the total requirement can be obtained at following prices, each delivered
to the factory:
Component
A
B
C
D

(`)
60
59
52
168

Second shift operations would increase direct wages by 25 per cent over the normal shift and fixed
overheads by ` 500 for each 1,000 (or part thereof) second shift hours worked.
You are required to present, with calculations:1.
Which component and in what quantities should be manufactured in the 20,000 hours of press
time available?
2.
Whether it would be profitable to make any of the balance of components required on a
second shift basis instead of buying them from outside suppliers.
(Nov 1992)
Q. 74. (Key Factor Make or Buy)
GG Ltd. manufactures and sells an equipment called water purifier. The cost data for each batch of
ten numbers of water purifier is as follows
Components
Machines Hours
Labour Hours
Variable costs
(`)
Fixed Costs as Apportioned (`)

A
20
64
36

B
28
108
52

C
24
116
64

D
4
24
26

E
2
8
22

Assembly costs (all variable) ` 50 per batch. Selling price is ` 800 per batch.
Maximum available machine capacity for making components A, B and C is 10,800 hours and it
cannot be increased further. Labour is available for making components D and E and for assembling
the product.
Estimated increase in demand next year is 50% and fixed costs in general may increase by ` 10,000.
In order to increase production capacity to meet increased market demand, the company decided to
purchase one of the machine made components.
Quote Ltd. is the only supplier of components A, B and C. Because of incomplete records, it is unable
to quote single figure prices. Its quotation is as follows:

40

Ideal Classes Final CA Advanced Management Accounting Volume 1


Component

A
B
C

Pessimistic
View
`
120
200
160

Probability

0.25
0.25
0.25

Most Likely
View
`
110
130
140

Probability

0.5
0.5
0.5

Optimistic
View
`
80
140
120

Probability

0.25
0.25
0.25

It is agreed between the companies that the price of each of the components will be determined on
an overall basis based on information found in the quotation.
You are required to:
1. Indicate in the context of key factor, the maximum number of batches that could be produced, if
each of the three alternatives namely buying A or B or C is considered.
2. Analyse the financial implication of purchase and advice which component is to be bought
keeping in view the fact that production capacity will be limited to a 50% increase.
3. Prepare a Profit Statement for the period assuming that the component chosen by you is bought
out and extra production is made and sold.
(May 2000)
Q. 75

(Key Factor Make or Buy)


A company manufactures three components. These components pass through two of the companys
departments P and Q. The machine hour capacity of each department is limited to 6,000 hours in a
month. The monthly demand for components and cost data are as under:
Components
Demand (units)

A
B
C
900
900
1,350
`
`
`
Direct material/unit
45
56
14
Direct labour/unit
36
38
24
Variable overheads/unit
18
20
12
Fixed overheads
P @ ` 8 per hour
16
16
12
Q @ ` 10 per hour
30
30
10
Total
145
160
72
Components A and C can be purchased from market at ` 129 each and ` 70 each respectively. You
are required to prepare a statement to show which of the components in what quantities should be
purchased to minimize the cost.
(Nov 2002)
Q. 76. (Key Factor Introduction of New Product)
XYZ Ltd is currently manufacturing 5000 units of the product XY 100 annually, making full use of its
machine capacity. The selling price and total costs per unit associated with XY 100 are as follows:

Selling price per unit


Cost per unit:
Direct material
Variable machine operating costs (` 100 per machine hour)
Manufacturing overhead costs
Marketing and administrative costs

(`)
900
200
150
180
200

Operating income per unit of 'XYZ 100'

730
170

XYZ Ltd can sell additional 3000 units of XY 100, if it can outsource additional units.

41

Ideal Classes Final CA Advanced Management Accounting Volume 1


ABC Ltd., a supplier of quality products, has agreed to supply upto 6,000 units of XY 100 per year at
a price of ` 650 per unit delivered at XYZs factory.
XYZ Ltd. can use its facility to produce an alternative product XY 200. It can sell up to 12,000 units
of XY 200 annually. Estimated selling price and total costs per unit to manufacture and sell 12,000
units of XY 200 are as follows:
(`)
Selling price per unit
600
Cost per unit:
Direct material
200
Variable machine operating costs (` 100 per machine hour)
50
Manufacturing overhead costs
60
Marketing and administrative costs
110
420
Operating income per unit of 'XYZ 200'
180
Other information pertaining to the operation of XYZ Ltd is as follows:
1. XYZ Ltd. use machine hours as the basis for assigning fixed manufacturing overhead. The fixed
manufacturing overhead for the current year is ` 3,00,000. These costs will not be affected by
the product-mix decision.
2. Variable marketing and administrative costs per unit for various products are as follows:
(`)
Manufactured
'XY 100'
80
Purchased
'XY 100'
40
Manufactured
'XY 200'
60
Fixed marketing and administrative costs for the current year is ` 6,00,000. These costs will not be
affected by the product-mix decision.
Calculate the quantity of each product that XYZ Ltd. should manufacture and/ or purchase to
maximize operating income. Show your calculations.
(May 2002)
Q. 77. (Key Factor Up-gradation of Machinery)
A firm has two machines namely, Machine P and Machine Q, Machine P can be used for the
production of either Product A or Product B or both. Machine Q can be used for the production
of either Product X or Product Y or both. In order to maintain customer relations a minimum
quantity of 1,500 units each of A and B and 1,200 units of X and Y should be produced by the
firm. The production and cost data for 2004 are as under:
Machine hours available:
P: 4,500 hours
Q: 5,100 hours
Products
A
B
X
Y
Machine used
P
P
Q
Q
Machine hour(s) required
per unit of output
1
1.25
1.25
0.8
`
`
`
`
Selling price
per unit
200
250
300
256
Direct materials
per unit
80
100
100
80
Direct labour
per machine hour
90
80
100
125
Variable overheads per machine hour
12
12
20
20
Fixed overheads ` 4 lakhs per annum. An additional expenditure involving a fixed overhead of `
25,000 per annum will convert the Machine P and Q into a versatile centre such that any four of
the products can be manufactured on these two machines. The rate of output on these machines
and the direct wage rate will however remain the same.

42

Ideal Classes Final CA Advanced Management Accounting Volume 1

Q.78.

Required:
1. Set an optimal product mix subject to the minimum market commitments both before and after
the conversion of the machines into a versatile centre.
2. Evaluate the profitability under the two sets of product mixes.
3. Advise the Management whether the conversion of machines should be undertaken or not.
(Nov 1984)
(Key Factor Minimum Supply Requirements)
A company, engaged in plantation activities, has 200 hectares of virgin land which can be used for
growing jointly or individually tea, coffee and cardamom. The yield per hectare of the different crops
and their selling price per Kg. are as under:
Yield (in Kg.)
Selling Price per Kg. (in `)
Tea
2,000
20
Coffee
500
40
Cardamom
100
250
The relevant cost data are given below:
a) Variable cost per Kg.
(in `)
Tea
Coffee
Cardamom
Labour charges
8
10
120
Packing materials
2
2
10
Other costs
4
1
20
Total cost
14
13
150
b) Fixed cost per annum
Cultivation and growing cost
Administration cost
Land revenue
Repair and maintenance
Other costs

` 10,00,000
` 2,00,000
` 50,000
` 2,50,000
` 3,00,000

Total fixed costs

` 18,00,000

The policy of the company is to produce and sell all the three kinds of products and the maximum
and minimum area to be cultivated per product is as follows:
Maximum (hectares)
Minimum (hectares)
Tea
160
120
Coffee
50
30
Cardamom
30
10
Calculate the most profitable product mix and the maximum profit which can be achieved.
Q.79.

(Selection of Supplier and Customers)


A Ltd. has set up a treatment plant at Surat. The company uses raw material X to convert into
finished product XA after treatment in its plant having a capacity to treat 60,000 tonnes of X per
annum. There is no loss of raw material in the treatment process. The variable costs of treatment
are ` 5 per tonne and the annual fixed costs amount to ` 7,50,000.
The company owns a fleet of vehicles to transport 260 lakhs tonnes kms. of X to the factory site.
The fixed costs of maintaining these vehicles amount to ` 10,40,000 per annum and the variable
costs amount to 8 paise per tonne-km. The company delivers XA to customers through another
transport agency at 15 paise per tonne-km., subject to a minimum annual payment of ` 1,25,000.
Facilities for hiring the transport for incoming of X and outgoing of XA are also available through
Global Transport Co. at a cost of 18 paise per tonne-km.
The company has three sources of procurement of raw material X, the relevant details being:

43

Ideal Classes Final CA Advanced Management Accounting Volume 1


Particulars
Quantity available
Distance from Surat
Price offered

Town A
9,000
300
90

(Tonnes p.a.)
(kms.)
(` per tonne)

Source of procurement
Town B
Town C
8,000
45,000
250
500
110
78

The company has demand of its finished product XA from the following customers:

Q.80.

Customers

Demand
(tonnes p.a.)

L
M
N
O
P
Q
R

6,000
6,000
15,000
9,000
10,000
9,000
10,000
65,000

Price
(per tonne)
`
200
188
170
150
192
220
200

Delivery terms

At Customer's site
Ex-factory Surat
Ex-factory Surat
Ex-factory Surat
At Customer's site
At Customer's site
At Customer's site

Distance from
Surat (kms)
9
22
14
25

You are required to recommend the best proposal for the purchase of raw material X , the break-up
of sale of finished product XA and transport plan for incoming and outgoing goods with a view to
maximize the profit. Also present a statement to show net profit as per above recommendation.
(Nov 1995)
BVX Ltd. Manufactures 3 garden furniture products chairs, benches and tables. The budgeted unit
cost and resource requirements of each of these items is detailed below:
Timber Cost
Direct Labour Cost
Variable Overhead Cost
Fixed Overhead Cost
Budgeted Volumes per annum

Chair
5
4
3
4.5
16.5
4,000

Bench
15
10
7.5
11.25
43.75
2,000

Table
10
8
6
9
33
1,500

These volumes are believed to equal the market demand for these products.
The fixed overhead costs are attributed to the 3 products on the basis of direct labour hours.
The labour rate is ` 4.00 per hour.
The cost of the timber is ` 2.00 per square metre.

The products are made from a special timber. A memo from the purchasing manager advices you
that because of a problem with the supplier, it is to be assumed that this special timber is limited in
supply to 20,000 square metres per annum. The sales director has already accepted an order for 500
chairs, 100 benches and 150 tables which if not supplied would incur a financial penalty of ` 2,000.
These quantities are included in the market demand estimates above.
The selling prices per unit of the 3 products are:
Chair
` 20
Bench
` 40
Table
` 50
Requirements:
1.
Determine the optimum production plan and state the net profit that this should yield per
annum.
2.
Calculate and explain the maximum prices which should be paid per square metre in order to
obtain extra supplies of the timber.

44

Ideal Classes Final CA Advanced Management Accounting Volume 1

Section 4 Miscellaneous
Q.81.

(Plant wise and Overall Profitability)


X Ltd. has two factories, one at Lucknow and another at Pune producing 7,200 tonnes and 10,800
tonnes of a product against the maximum production capacity of 9,000 and 11,880 tonnes
respectively at Lucknow and Pune.
10% of the raw material introduced is lost in the production process. The maximum quantities of raw
material, available locally are 6,000 and 13,000 tonnes at ` 720 and ` 729 per tonne at Lucknow and
Pune respectively. For the additional needs a supplier of Bhopal is ready to supply raw material at
our factory site at ` 792 per tonne.
Other variable costs of the production process are `. 22.32 lacs and ` 32.94 lacs and fixed costs are `
18 lacs and ` 24.84 lacs respectively for Lucknow and Pune.
The output is sold at a selling price of ` 1,450 and ` 1,460 per tonne by Lucknow and Pune factory
respectively.
You are required to compute net profit earned in respect of each factory.
Can you suggest any other alternative production plan for both the factories without any change in
present total output of 18,000 tonnes whereby the company may earn optimum profit.

Q. 82. (Shut Down or Continue)


A paint manufacturing company manufactures 2,00,000 per annum medium sized tins of Spray Lac
Paints when working at normal capacity. It incurs the following costs of manufacturing per unit:
(`)
Direct material
7.8
Direct labour
2.1
Variable overhead
2.5
Fixed overhead
4.00
Product cost (per unit)
16.40
Each unit (tin) of the product is sold for ` 21 with variable selling and administrative expenses of 60
paise per tin.
During the next quarter only 10,000 units can be produced and sold. Management plans to shut
down the plant estimating that the fixed manufacturing cost can be reduced to ` 74,000 for the
quarter.
When the plant is operating, the fixed overheads are incurred at a uniform rate throughout the year.
Additional costs of plant shut-down for the quarter are estimated at ` 14,000.
You are required:
1. To express your opinion, along with the calculations, as to whether the plant should be shutdown during the quarter, and
2. To calculate the shut-down point for quarter in units of products (i.e., in terms of number of
tins.)
(May 1991)
Q. 83. (Effect of change in Plant Layout)
Jai Textiles Ltd. has been earning low profits. A special task force appointed for reviewing
performance and prospects reports the following:
The company has 1,200 looms working 2 shifts per day. There are 25 sections of 48 looms each. Each
section has 24 weavers and a jobber. Thus there are 1,250 direct labourers, other than indirect
labourers and service hands. The working time is between 7 a.m. and 12 mid-night, comprising 2
shifts of 8 hours each, with half hour interval between shifts. The production is 18 lakh meters per
month and the sales realisation is ` 3 per metre. The average wage of the direct labourer is ` 800

45

Ideal Classes Final CA Advanced Management Accounting Volume 1


per month and the fixed costs amount to ` 1,75,000 per month. The product cost is ` 2.25 per metre
in addition to direct wages.
The following suggestions are to be considered:
1. Labour productivity can be improved by changing the layout of the machines.
2. Given the space available, with the proposed change in layout, only 1,008 looms can be reinstalled, with 48 looms in each section.
3. Technically, a section of 48 looms can be run with 12 weavers, a helper and a jobber. It will be
necessary to increase the wage of direct labour, for such sections, by ` 110 per head per month.
There will be some drop in production per loom. The company is not for retrenchment of labour.
4. The company can run a third shift between 12 mid-might and 7a.m., with a half an hour interval.
However, for the six and half hours' work, eight hours' wage will have to be paid.
5. Only 18 lakh meters can be sold at the present price of ` 3 per metre. There is an export offer for
4.5 lakh meters at ` 2.70 per metre.
6. As an initial step, the company can switch to 3 shifts working, with 12 sections having 25 direct
labourers each and 9 sections having 14 direct labourers each. Progressive conversion to 14
heads per section, for all sections, can be planned, as direct labourers retire or voluntarily leave
the job. The production, with three shift working will be 22.5 lakh meters. Additions to fixed
costs will amount to ` 50,000 per month.
Examine the implications of the proposals for the companys profit and give your advice.
(Nov 1991)
Q.84.

(Selection of Supplier and Customers)


A manufacturing company produces a chemical product which passes through two processes factory
and finishing. It has the capacity to process an input of 1,00,000 kgs of raw material. Normal scrap
will be 10% and 5% of input in factory and finishing process respectively. The realizable value of such
scrap is ` 4 and ` 8 per kg respectively for factory and finishing process to be credited against the
cost of respective process.
Relevant cost data for the coming year are:
(`)
Particulars
Factory Process
Finishing Process
Direct Wages
6,00,000
5,50,000
Overheads
2,28,000
4,22,900
There are three possible sources of purchase of raw materials:
Supplier
X
Y

Purchase price per kg


`
5.00
5.60

5.30

Maximum quantity
Kgs.
60,000
80,000
Provided entire quantity of 1,00,000 kgs. is
ordered, otherwise at `5.80 per kg.

In each case, the company is required to collect the raw materials from the godown of supplier.
Variable transport cost depends upon the distance involved. The same are, as under:
Supplier
Transport cost (per kg.)

X
30 paise

Y
25 paise

Z
25 paise

Fixed transport cost would be ` 1,00,000 per annum irrespective of the supplier to be contacted. The
output of the finishing process can be sold to three prospective customers, their offer being as
follows:

46

Ideal Classes Final CA Advanced Management Accounting Volume 1


Customer
A
B
C

Price per kg of output


(`)
32.50
32.00
30.90

Trade discount
(%)
2
2
-

Conditions
Maximum quantity 40,000 kgs
Maximum quantity 80,000 kgs
Provided the entire output is sold to him

In case of supplies to customers A and B, the fixed delivery costs will be ` 1,500 per month and the
variable delivery costs will be 65 paise and 36 paise per kg. respectively. Customer C will collect the
entire output from the warehouse of the company.
You are required to indicate with reasoning:
1. Choice of supplier with comparative cost tables.
2. Choice of customer with comparative tables of net realizations.
Also prepare the statements showing process costs and overall results.
(May 1993)
Q.85.

(Sub Contracting)
A company manufactures two products EXE and WYE, which pass through two of its departments
exclusively used for them. A market research study conducted by the company reveals that the
company can sell either 38,500 units of EXE or 31,500 units of WYE in a year. The manufacturing cost
and selling price details are as under:
EXE
WYE
`
`
Selling price per unit
375
540
Costs per unit:
Department 1:
Direct materials
58
100
Direct labour
5 hour
50
7.5 hour
75
Department 2:
Direct materials
21
26
Direct labour
7.5 hour
90
10 hours 120
Overheads:
Department 1
Department 2
Variable overhead rate per direct labour hour
` 2.40
` 3.60
Fixed overheads
` 5,00,000
` 10,00,000
Budgeted direct labour hours
1,75,000
2,80,000
Since the quantity which can be sold exceeded the production capacity, the company has been
considering the use of sub-contracting production facilities. Accordingly, when tenders were floated,
two contractors responded as under:
Contractor DS offers to produce upto a maximum of 17,500 units of EXE or 14,000 units of WYE in a
year for the type of work done by department 1 of the company. The price charged by DS is ` 138
per unit of EXE and ` 212 per unit of WYE. These prices included the cost of direct materials used in
department 1 of the company.
Contractor DW can produce upto a maximum of 11,200 units of EXE or 7,000 units of WYE in a year
for the type of work done by department 2 of the company. The price charged by DW is ` 150 per
unit of EXE and ` 192 per unit of WYE. These prices included the cost of direct materials used in
department 2 of the company.
Required:
1. If the company does not wish to use the sub-contracting facility, which of the two products and
in what quantity should be produced and sold by the company by using its own manufacturing
capacity to earn maximum profit? Calculate the resultant maximum profit.

47

Ideal Classes Final CA Advanced Management Accounting Volume 1


2. If the company wishes to produce either 38,500 units of EXE or 31,500 units of WYE by using
sub-contracting facility, state which of the two products should be produced to maximize the
profits. Calculate the resultant maximum profit?
(May 2003)
Q.86.

(Captive Consumption)
A company manufacturing chemicals furnishes the following data of their activities for year 2003-04.
The company manufactures three products namely Ethelene, EDC and VCL. Ethelene is consumed for
making EDC and EDC is consumed for making VCL. One metric ton of Ethelene is required to make
one metric ton of EDC and one metric ton of EDC is required for making one metric ton of VCL. The
other particulars are:
Particulars
Production capacity Per annum (Metric Tons)
Cost Per Metric Ton:
Variable Costs
(`)
Product Fixed Costs
(`)
Common Fixed Costs
(`)
Total
Selling Price Per Metric Ton
(`)
Sales Per Annum
(Metric Tons)

Ethelene
25,000

EDC
30,000

VCL
30,000

20
20
10
50
-

30
30
15
75
150
10,000

40
40
20
100
300
15,000

The company restricts the manufacture of all its products only to the extent of the sales demand.
The management is concerned with the low capacity utilization. In order to achieve fuller utilization
of the plant capacity, the company entered into negotiations with various parties. As a result of the
negotiations, X who buys one-third of the current sales volume of VCL, offers to buy 20,000 metric
tons of VCL per annum at ` 250 per metric ton provided the entire quantity of 20,000 metric ton is
sold to him. This purchase is for captive consumption of X and therefore will not effect the market
price of VCL. X also offers to supply EDC for manufacture of VCL to the extent of 5,000 metric tons at
a price of ` 125 per metric ton. The company can also buy EDC from the open market at ` 140 per
metric ton if the order is for 10,000 metric tons or more:
The bases of various costs given above are as follows:
1. Variable costs exclude the cost of internally consumed Ethelene consumed in the manufacture
of EDC and costs of EDC consumed in the manufacture of VCL.
2. Fixed costs are based on normal capacity production.
3. The product fixed costs can be avoided only if there is nil production of the product concerned.
4. Common fixed costs are to be incurred irrespective of production and sales.
5. No closing stocks are maintained.
You are required to:
(a) Draw up a statement of profitability in respect of the year 2003-04 as originally envisaged by
the company.
(b) If the company decides to accept the offer of X to buy 20,000 metric tons of VCL at ` 250 per
metric ton and if the balance quantity of production of VCL can be sold in the market, show
the revised statement of profitability of the company.
(Nov 1994)

Practice Section
Q. 87. A businessman employees 20 swing machinists, but he is aware that ten are the better workers than
others. He is considering to conduct a training programme for his ten less efficient mechanists to

48

Ideal Classes Final CA Advanced Management Accounting Volume 1


increase their efficiency to be equal to that achieved by better workers. Relevant data are as
follows;
There is one sewing machine for each machinist.
All the machinists are engaged on similar work and are paid ` 2.20 each good garment produced
on piece work system.
To rectify each rejected garment costs ` 4, this work is done by subcontractor.
Garment machining department operates 2,000 hours a year.
Average output of per machinist (on the basis of all 20 machinists) is 12 good garments with one
rejected per worker per hour. However 10 less efficient machinists averages only 10 good
garments with 1.5 rejected per worker per hour.
Depreciation of each sewing machine is ` 10, 000 per year and the variable cost of power,
cleaning and preventive maintenance is ` 5 per hour per machine.
Fixed production overhead other than depreciation is ` 20 per machine hour.
Selling price per garment is ` 18.
Direct material cost per garment is ` 12.
Training will not reduce productive hours.
There is no problem in selling increased output.
You are required
1. To prepare a statement of comparative costs for the better worker and the less efficient
workers excluding material costs.
2. To find out the benefit derived over a one year period, if ` 1,00,000 is spent on a training course
for the less efficient workers to match the efficiency with the better workers.
(Nov 05, 11 Marks)
Q.87.

X Limited having an installed capacity of one lakh units of a product is currently operating at 70 per
cent utilization. At current level of input prices, the F.O.B. costs per unit, taking credit for applicable
export incentive workout as follows:
Capacity Utilisation
FOB cost per unit (`)

70%
97

80%
92

90%
87

100%
82

The company has received three Foreign offers as under:


Source A: 5,000 units @ ` 55 per unit FOB
Source B: 10,000 units @ `. 52 per unit FOB
Source C: 10,000 units @ ` 51 per unit FOB
Required:
Advice the company whether it should accept any or all of the export orders.
(Nov 07, 10 Marks)
Q.88.

TQM Limited makes engines for motor cars for its parent company and for two other motor car
manufacturers.
On 31st December, the company has sufficient work order for January and one further order for
21,000 engines. Due to recession in the economy, no further order are expected until May when it is
hoped economic prospect for the motor car industry will have improved. Recently factory has been
working at only 75% of full capacity and the order for 21,000 engines represents about one month
production at this level of activity.
The board of directors are currently considering following two options:
1. Complete the order in February and close the factory in March and April.
OR
2. Operate at 25 per cent of full capacity for each of three months of February, March and April.

49

Ideal Classes Final CA Advanced Management Accounting Volume 1


The costs per month at different levels of activities are as follows:
At 75% (`)
Direct Material
5,25,000
Direct Labour
5,23,600
Factory overhead:
Indirect material
8,400
Indirect labour
1,01,500
Indirect expenses:
Repairs and maintenance
28,000
Others expenses
52,500
Office overheads:
Staff salaries
1,48,400
Other overheads
28,000

At 25% (`)
1,75,000
1,73,250

Idle (`)
---

4,900
59,500

4,900
--

28,000
34,300

-26,600

98,000
19,950

67,550
11,200

Other information is as follows:


Material cost and labour cost will not be incurred where there is no production.
On the reopening of the factory, one time cost of training and engagement of new personnel
would be ` 65,800 and overhauling cost of plant would be ` 14,000.
Parent company can purchase engines from open market at reasonable price.
Required:
1. To express your opinion, along with calculations, as to whether the plant should be shut down
during the month of March and April or operate 25% of full capacity for three months.
2. To list and comment on cost and non-costs factors which might to relevant to the discussion.
(June 09, 11 Marks)
Q.89.

Fairbilt Furniture Ltd. manufactures three products: Tables, Chairs and Cabinets. The company is in
the process of finalizing the plans for the coming year; hence the executives thought it would be
prudent to have a look at the product-wise performance during the current year. The following
information is furnished:
Tables
Chairs
Cabinets
Unit selling price
80
60
36
Direct material
28
24
16
Direct labour
20
12
12
Factory overheads:
Variable
8
6
4
Fixed
8
6
1.28
Cost of production
64
48
33.28
Selling, distribution and general
administration expenses:
Variable
4
2
2
Fixed
4
6
1.52
Unit cost (I)
72
56
36.80
Unit profit (loss) (II)
8
4
(0.80)
Sales volume (units)
10,000
15,000
15,000
Profit (loss)
80,000
60,000
(12,000)
For the coming period, the selling prices and the cost of three products are expected to remain
unchanged. There will be an increase in the sales of tables by 1,000 units and the increase in sales of
cabinets is expected to be 8,000 units. The sales of chairs will remain to be unchanged. Sufficient
additional capacity exists to enable the increased demands to be met without incurring additional
fixed costs. Some among the executives contend that it will be unwise to go for additional
production and sale of cabinets, since it is already making losses at ` 0.80 per unit. The suggestion is
that cabinets should be eliminated altogether.

50

Ideal Classes Final CA Advanced Management Accounting Volume 1


Do you agree? Substantiate with necessary analysis and determine the product wise and overall
profits for the coming year.
(June 09, 10 Marks)
Q.90.

X is a multiple product manufacturer. One product line consists of motors and the company produces
three different models. X is currently considering a proposal from a supplier who wants to sell the
company blades for the motors line.

The company currently produces all the blades it requires. In order to meet customer's
needs, X currently produces three different blades for each motor model (nine different
blades).
The supplier would charge `25 per blade, regardless of blade type. For the next year X has
projected the costs of its own blade production as follows (based on projected volume of
10,000 units):
Direct materials
` 75,000
Direct labour
` 65,000
Variable overhead
` 55,000
Fixed overhead:
Factory supervision
` 35,000
Other fixed cost
` 65,000
Total production costs
` 2,95,000
Assume (1) the equipment utilized to produce the blades has no alternative use and no B
market value, (2) the space occupied by blade production will remain idle if the company
purchases rather than makes the blades, and (3) factory supervision costs reflect the salary
of a production supervisor who would be dismissed from the firm if blade production
ceased.
1. Determine the net profit or loss of purchasing (rather than manufacturing), the blades required
for motor production in the next year.
2. Determine the level of motor production where X would be indifferent between buying and
producing the blades. If the future volume level were predicted to decrease, would that
influence the decision?
3. For this part only, assume that the space presently occupied by blade production could be leased
to another firm for ` 45,000 per year. How would this affect the make or buy decision?
(June 09, 9 Marks)
Q.91.

A company has prepared the following budget for the forthcoming year:
` lakhs
20.00
3.60
6.40

Sales
Direct materials
Direct labour
Factory overheads:
Variable
Fixed
Administration overheads
Sales commission
Fixed selling overheads
Total costs
Profit

2.20
2.60
1.80
1.00
0.40
18.00
2.00

51

Ideal Classes Final CA Advanced Management Accounting Volume 1


The policy of the company in fixing selling prices is to charge all overheads other than the prime
costs on the basis of percentage of direct wages and to add a markup of one ninth of total costs for
profit.
While the company is confident of achieving the budget drawn up as above, a new customer
approached the company directly for execution of a special order. The direct materials and direct
labour costs of the special order are estimated respectively at ` 36,000 and ` 64,000. This special
order is in excess of the budgeted sales as envisaged above. The company submitted a quotation of
` 2,00,000 for the special order based on its policy. The new customer is willing to pay a price of `
1,50,000 for the special order. The company is hesitant to accept the order below total cost as,
according to the company management, it will lead to a loss.
You are required to state your arguments and advise the management on the acceptance of the
special order.
(Nov 08, 7 Marks)
Q.92.

Paints Ltd. manufactures 2,00,000 tins of paint at normal capacity. It incurs the following
manufacturing costs per unit:
`
Direct material
7.80
Direct labour
2.10
Variable overhead
2.50
Fixed overhead
4.00
Production cost / unit
16.40
Each unit is sold for ` 21, with an additional variable selling overhead incurred at ` 0.60 per unit.
During the next quarter, only 10,000 units can be produced and sold. Management plans to shut
down the plant estimating that the fixed manufacturing cost can be reduced to ` 74,000 for the
quarter.
When the plant is operating, the fixed overheads are incurred at a uniform rate throughout the year.
Additional costs of plant shut down for the quarter are estimated at ` 14,000.
You are required:
1. To advise whether it is more economical to shut down the plant during the quarter rather than
operate the plant.
2. Calculate the shut down point for the quarter in terms of numbering units.
(Nov 08, 6 Marks)

Q.93.

PQ Ltd. makes and sells a labour-intensive product. Its labour force has a learning rate of 80%,
applicable only to direct labour and not to variable overhead.
The cost per unit of the first product is as follows:
Direct materials
10,000
Direct labour
8,000 (@ `4 per hour)
Variable overhead
2,000
Total variable cost
20,000

PQ Ltd. has received an order from X Ltd. for 4 units of the product. Another customer, Y Ltd. is also
interested in purchasing 4 units of the product. PQ Ltd. has the capacity to fulfill both the orders. Y
Ltd. presently purchases this product in the market for `17,200 and is willing to pay this price per
unit of PQ's product. But X Ltd. lets PQ choose one of the following options:
1. A price of `16,500 per unit for the 4 units it proposes to take from PQ.
Or

52

Ideal Classes Final CA Advanced Management Accounting Volume 1


2. Supply X Ltd.'s idle labour force to PQ, for only 4 units of production, with PQ having to pay only
Re. 1 per labour hour to X Ltd.'s workers. X Ltd.'s workers will be withdrawn after the first 4 units
are produced. In this case, PQ need not use its labour for producing X Ltd.'s requirement. X Ltd.
assures PQ that its labour force also has a learning rate of 80%. In this option, X Ltd. offers to buy
the product from PQ at only `14,000 per unit.
X and Y shall not know of each other's offer.
If both orders came before any work started, what is the best option that PQ may choose?
Present suitable calculations in favour of your argument.
(June 09, 8 Marks)
Q.94.

Ret Ltd., a retail store buys computers from Comp Ltd. and sells them in retail. Comp Ltd. pays Ret
Ltd. a commission of 10% on the selling price at which Ret sells to the outside market. This
commission is paid at the end of the month in which Ret Ltd. submits a bill for the commission. Ret
Ltd. sells the computers to its customers at its store at `30,000 per piece Comp Ltd. has a policy of
not taking back computers once dispatched from its factory. Comp Ltd. sells a minimum of 100
computers to its customers. Comp Ltd. charges prices to Ret Ltd. as follows:
` 29,000 per unit, for order quantity 100 units to 140 units
` 26,000 per unit, for the entire order, if the quantity is 141 to 200 units. Ret Ltd. Cannot order less
than 100 or more than 200 units from Comp Ltd.
Due to the economic recession, Ret Ltd. will be forced to offer as a free gift, a digital camera costing
it `4,500 per piece, which is compatible with the computer. These cameras are sold by another Co.,
Photo Ltd. only in boxes, where each box contains 50 units. Ret Ltd. can order the cameras only in
boxes and these cameras cannot be sold without the computer.
In its own store, Ret Ltd. can sell 110 units of the computer. At another far of location, Ret Ltd. can
sell upto 80 units of the computer (along with its free camera), provided it is willing to spend `5,000
per unit on shipping costs. In this market also, the selling price that each unit will fetch is `30,000
per unit.
You are required to:
1. State what is Ret's best strategy along with supporting calculations.
2. Compute the break-even point in units, considering only the above costs.
(June 09, 13 Marks)

Q.95. B Ltd. makes industrial power drills, which is made by the use of two components A (electrical and
mechanical components and B (plastic housing). The following table shows the cost of plastic
housing separately from the cost of the electrical and mechanical components:
A
B
A&B
Electrical and
Plastic Housing
Industrial Drills
Mechanical
Components

`
Sales 1,00,000 units @ `100
Variable Costs:
Direct materials
Direct Labour
Variable factory overhead
Other Variable Costs
Sales commission @10% of sales
Total variable costs
Contribution
Total fixed costs
Operating income

`
1,00,00,000

44,00,000
4,00,000
1,00,000
1,00,000
10,00,000
60,00,000
22,20,000

53

5,00,000
3,00,000
2,00,000
10,00,000
4,80,000

49,00,000
7,00,000
3,00,000
1,00,000
10,00,000
70,00,000
30,00,000
27,00,000
3,00,000

Ideal Classes Final CA Advanced Management Accounting Volume 1


Answer the following questions independently:
1. During the year, a prospective customer offered `82, 000 for 1,000 drills. The drills would be
manufactured in addition to the 1,00,000 units sold. B Ltd. would pay the regular sales
commission rate on the 1,000 drills. The Chairman rejected the order because it was below our
costs. Calculate operating income if B Ltd. accepts the offer.
2. A supplier offers to manufacture the yearly supply of 1,00,000 units plastic housing components
for `13.50 each. Assume that B Ltd. would avoid `3,50,000 of the costs assigned to plastic
housing if it purchases. Calculate operating income if B Ltd. decides to purchase the plastic
housing from the supplier.
3. Assuming that B Ltd. could purchase 1,20,000 units (plastic housing components) for `13.50
each and use the vacated plant capacity for the manufacture of deluxe version of drill of 20,000
units (and sell them for `130 each in addition to the sales of the 1,00,000 regular units) at a
variable cost of `90 each, exclusive of housings and exclusive of the 10% sales commission. All
the fixed costs pertaining to the plastic housing would continue, because these costs are related
to the manufacturing facilities primarily used. Calculate operating income of B Ltd. purchases the
plastic housings and manufacture the deluxe version of drills.
(Nov 09, 8 Marks)
Q.96.

Spares Ltd. produces spare part X for cars. The company has an annual production capacity of
1,80,000 units of X. However, the actual production is carried out according to the volume of order
received. For the next year, the company has received an order for a value of ` 64,00,000. To meet
the requirements of the order, the company has to work at 70% capacity for the first four months,
80% capacity for next six months, and 90% capacity for the remaining period of the year. Assume no
opening or closing stocks.
The following information is available:
Material cost is ` 15 per unit.
Labour ` 12 per unit, subject to a minimum of ` 1,30,000 p.m.
Variable overheads ` 5 per unit.
Fixed overheads ` 16,000 per month.
Semi-variable overheads ` 75,000 per annum incurred upto 70% average annual capacity utilisation.
Thereafter, it increases at ` 5,000 per annum for every 10% average annual capacity increase.
If the company targets a return of 27% on budgeted cost, should the order be accepted?
Justify your answer showing budgeted annual values for each element of cost for the next year.
(May 2010, 12 Marks)

Q.97. E Ltd. is engaged in the manufacturing of three products in its factory. The following budget estimates
are prepared for 2009-10:
Products
A
10,000
40
10
8
8
16
-2

Sales (Units)
Selling price per unit (`)
Direct material per unit (`)
Direct wages per unit @ ` 2 per hour
Variable overhead per unit (`)
Fixed Overhead per unit (`)
Profit/Loss

B
25,000
75
14
12
9
18
22

C
20,000
85
18
10
10
20
27

After the finalization of the above manufacturing schedule, it is observed that presently only 80%
capacity being utilised by these three products. The production activities are made at the same

54

Ideal Classes Final CA Advanced Management Accounting Volume 1


platform and it may be interchangeable among products according to requirement. In order to
improve the profitability of the company the following three proposals are put for consideration:
1. Discontinue product A and capacity released may be used for either product B or C or equally
shared. The fixed cost of product A is avoidable. Expected changes in material cost and selling
price subject to the utilisation of product As capacity are as under:
Product B: Material cost increased by 10% and selling price reduced by 2%.
Product C: Material cost increased by 5% and selling price reduced by 5%.
2. Discontinue product A and divert the capacity so released and the idle capacity to produce a
new product D for meeting export demand whose per unit cost data are as follows:
`
Selling price
60
Direct Material
28
Direct wages @ ` 3 per hour
12
Variable overhead
6
Fixed Cost (Total)
1,05,500
3. Product A, B and C are continuously run and hire out the idle capacity fixing a price in such a way
that the same rate of profit per direct labour hour is obtained in the original budget estimates.
Required:
1. Prepare a statement of profitability of Products A, B and C in existing situation.
2. Evaluate the above proposals independently and calculate the overall profitability of the
company under each proposal.
3. What proposal should be accepted, if the company wants to maximise its profit?
(May 2010, 10 Marks)
Q.98.

The selling price per unit of a product is ` 14. For the forthcoming, period, the demand will be only
5,000 units. The fixed expenses at 50% activity (5,000 units) will be ` 30,000. The company is
thinking of shutting down operations, in which case an additional amount of ` 2,000 will have to be
incurred for shutting down and only 20,000 of the above fixed costs can be avoided.
What should be the variable cost per unit to recommend a shut down?
(Nov 2010, 10 Marks)

Q.99. G Ltd. produces and sells 95,000 units of X in a year at its 80% production capacity. The selling price
of product is ` 8 per unit. The variable cost is 75% of sales price per unit. The fixed cost is ` 3,50,000.
The company is continuously incurring losses and management plans to shut-down the plant. The
fixed cost is expected to be reduced to ` 1,30,000. Additional costs of plant shut-down are expected
at ` 15,000.
Should the plant be shut-down? What is the capacity level of production of shut-down point?
(Nov 2010, 5 Marks)

Home Work Section


Q.100. (Simple Decision Making)
ABC Ltd. produces a variety of products each having a number of component parts. Product B takes
5 hours to produce on a particular machine which is working at full capacity. B has a selling price of `
100 and variable cost of ` 60 per unit. A component part X-100 could be made on the same machine
in two hours at a variable cost of ` 10 per unit. The suppliers price for the component is ` 25 per
unit.

55

Ideal Classes Final CA Advanced Management Accounting Volume 1


Required: Advise whether the company should buy the component X-100.
(If necessary make suitable assumptions)
(May 1995)
Q.101. (Simple Decision Making)
Product A takes five hours to produce on a particular machine and it has a selling price of ` 50 and
a marginal cost of ` 35. On the same machine, another Product B can be made at two hours at a
marginal cost of ` 5 per unit. Suppliers price of product B is ` 10 per unit.
Assuming that machine hour is the key factor, advice whether Product B could be bought out or
manufactured.
(Nov 1999)
Q.102. (Simple Decision Making)
A machine manufactures 10,000 units of a part at a total cost of ` 21 of which ` 18 is variable. This
part is readily available in the market at ` 19 per unit.
If the part is purchased from the market then the machine can either be utilized to manufacture a
component in same quantity contributing ` 2 per component or it can be hired out at ` 21,000.
Recommend which of the alternative is profitable?
(May 1997)
Q.103. (Simple Decision Making)
Unique Products manufactures and sells in a year 20,000 units of a particular product to definite
customers at a price of ` 100 per unit. The concern has a capacity to produce 25,000 units of the
product per annum. To produce beyond 25,000 units per annum, the concern will have to install a
new equipment at a cost of ` 15 lakhs. The equipment will have a life span of 10 years and will have
no residual value. There is an offer from a client to purchase 10,000 units of the product regularly at
a price of ` 90 per unit. The order, if accepted, will have to be over and above the existing level of
production of 20,000 units.
The cost structure is as under:
(`/unit)
Direct Material
30
Direct Labour
20
Variable Overhead
10
Profit
20
During the coming year, it has been estimated that the cost of direct material, as compared to the
current year, will increase by 10%. Because of the certain wage agreement direct labour cost will
increase by 25%. Fixed overheads will increase by 10%. If the new order for 10,000 units is accepted,
fixed overheads will increase further by ` 60,000 due to increased administrative charges.
You are required to analyse whether the concern should accept the order or instead of that try to
secure order for the balance unused capacity, as available now, through some sales promotion
expenses which will be ` 50,000 per annum. Ignore financial charges for the new investment.
(May 2000)
Q.104. (Simple Decision Making)
Paramount Food Products is a new entrant in the market for chocolates. It has introduced a new
product - Sweetie. This is a small rectangular chocolate bar. The bars are wrapped in aluminum foil
and packed in attractive cartons containing 50 bars. A carton, is therefore, considered the basic sales
unit. Although management had made detailed estimates of costs and volumes prior to undertaking
this venture, new projections based on actual cost experience are now required.
Income Statements for the last two quarters are each thought to be representative of the costs and
productive efficiency we can expect in the next few quarters. There were virtually no inventories on
hand at the end of each quarter. The income statements reveal the following:-

56

Ideal Classes Final CA Advanced Management Accounting Volume 1


Particulars

First Quarter

Second Quarter

Sales
(50,000 x ` 24)
(70,000 x ` 24)
Cost of goods sold
Gross margin

12,00,000
7,00,000
5,00,000

16,80,000
8,80,000
8,00,000

Selling and administration


Net income (loss) before taxes
Tax (negative)
Net income (loss)

6,50,000
(1,50,000)
(60,000)
(90,000)

6,90,000
1,10,000
44,000
66,000

The firm's overall marginal and average income tax rate is 40%. This 40% figure has been used to
estimate the tax liability arising from the chocolate operations.
Required:
1. Management would like to know the break-even point in terms of quarterly carton sales for the
chocolates.
2. Management estimates that there is an investment of ` 30,00,000 in this product line. What
quarterly carton sales and total revenue are required in each quarter to earn an after-tax return
of 20% per annum on investment?
3. The firm's marketing people predict that if the selling price is reduced by ` 1.50 per carton (`
0.03 off per chocolate bar) and a ` 1,50,000 advertising campaign among school children is
mounted, sales will increase by 20% over the second quarter sales.
Should the plan be implemented?
(Nov 1990)
Q.105. (Simple Decision Making)
A firm furnishes the following information:
Capacity in Units
2,000
3,000
4,000
5,000
6,000

Unit Cost `
40
35
34
32
31

Unit Price `
100
95
94
-

At present the firm is operating at 4,000 units capacity and has received an order for 2,000 units
from an export market at ` 28 per unit. Should the order be accepted?
(May 2000)
Q.106. (Simple Decision Making)
A company can produce and sell at its maximum capacity 20,000 units of a product. The sale price is
` 100. The present sales is 15,000 units. To produce over 20,000 units and upto another 10,000 units
some balancing equipments are to be installed at a cost of ` 10 lakhs and the same will have a life
span of 10 years.
The current cost structure is as under:
Direct Material
Direct Labour
Variable Overhead
Profit

30% of the sale value


20% of the sale value
` 20 per unit
` 15 per unit

The present cost is estimated to go up due to price escalation as under:


10% in Direct Material from present level of 30%.

57

Ideal Classes Final CA Advanced Management Accounting Volume 1


25% in Direct Labour from present level of 20%.
` 50,000 in fixed overhead per year.
There is a concrete proposal from a party to take 10,000 units additionally over the present level of
output, on a long term basis at a unit price of ` 90. Apart from the investment of ` 10 lakhs, as
shown above, the fixed overheads will increase by ` 50,000 due to additional administrative
expenses.
The company is in a dilemma as to whether to accept the order for 10,000 units or to use the
present unused capacity of 5,000 units for which there will be additional selling expenditure of `
50,000.
Ignore financing charges and give your recommendation.
(Nov 1996)
Q.107. (Plant wise and Overall Profitability)
Dinesh Dairies Ltd. has two processing and bottling plants, Danida and Danima, in adjoining districts.
The comparatives cost and revenue data budgeted per month are as follows:
Particulars
Bottles
Closures
Crates
Milk loss
Electricity
Fuel
Water
Total Variable Costs
Electricity
Salaries and wages
Depreciation
Total Fixed Cost
Total Costs
Sales realisation
Profit

Danida
(1,00,000 litres)
1,00,000
90,000
14,000
30,000
14,000
40,000
10,000
2,98,000
13,500
90,000
50,000
1,53,500
4,51,500
7,00,000
2,48,500

Danima
(75,000 litres)
79,000
71,500
12,500
47,000
14,000
46,000
11,250
2,81,250
11,000
60,000
20,000
91,000
3,72,250
5,25,000
1,52,750

Danima's high cost, low margin status draws management's attention. It is also observed that Danida
can increase its production by 50 percent with the existing plant capacity and without additional
manpower. Two proposals are under consideration:
1.
Cut down Danima's production by 25,000 litres and increase Danida's production by 25,000
litres.
2.
Cut down Danima's production by 50,000 litres and increase Danida's production by 50,000
litres.
For the additional quantity produced in excess of 1,00,000 litres, Danida will incur ` 0.40 per litre
towards group incentive. Transporting the additional output from Danida to Damima's region for
sale will cost ` 10,000 in both cases.
a.
Prepare a statement to show the contribution and the profit for Danida, Danima and for the
company as a whole, for each proposal. Comment on the results.
b.
Management is keen that the cut in Danima's production should not result in reporting loss, as
it would demoralize its employees. If break-even production is to be maintained in Danima
and the balance alone is to be transferred to Danida, show the contribution and the profit for
Danida, Danima and the company as a whole.
(Nov 1991)

58

Ideal Classes Final CA Advanced Management Accounting Volume 1


Q. 108. (Plant wise and Overall Profitability)
A company has two plants one at Sambalpur and the other at Bilaspur, where production of goods
takes place.
The basic raw material requirement is 80% of the finished product, by weight. Such materials are
available locally, but are limited to 6,000 M.T. at ` 1,800 per M.T. at Sambalpur and 16,000 M.T. at `
2,000 per M.T. at Bilaspur. Any extra requirements, will have to be procured from Jamshedpur at `
2,500 per M.T. Other details are as under:
Particulars

For unit at
Sambalpur
12,000
80
156
108

For unit at
Bilaspur
15,000
60
192
120

Annual output
(M.T.)
Capacity utilisation
(%)
Other variables
(` lakhs)
Fixed cost
(` lakhs)
You are required to determine:
(a) The cost break-up of each unit per M.T. of output.
(b) The quantity of production at each unit from the availability of local supplies of basic raw
material only, by keeping the same total production of the company, as a whole.
(c) Cost savings, if any, as per the revised schedule of production.
(Nov 2001)
Q. 109. (Plant wise and Overall Profitability)
Sweet Dreams Ltd. manufactures and markets three products A, B and C in the State of Haryana and
Rajasthan. At the end of first half of 2004-05 the following absorption based profit statement has
been drawn by the accountant:
(` '000)
Particulars
Haryana
Rajasthan
Total
Sales
3,000
900
3,900
Manufacturing Cost of Sales
2,331
699
3,030
Gross Profit
669
201
870
Administration Expenses (i)
120
36
156
Selling Expenses
(ii)
184
169
353
Total Expenses
304
205
509
Net Profit
365
(4)
361
1. The expenses are constant and common to both the States. They stand allocated on the basis of
sales.
2. The expenses are semi-fixed but specifically relate to the respective State.
The management is worried to note that the decision taken to market the products in Rajasthan to
utilize the idle capacity has proved wrong and wish to cover only Haryana State.
The incharge marketing division is not satisfied with the above way of profit presentation. He is of
the firm opinion that sales affected in the State of Rajasthan is contributing profits. For the next half
year he expects no increase in demand in Haryana while for Rajasthan he anticipates to sell B or C
more by 50% of existing sales. This will utilize the idle capacity in full.
The product wise relevant details for the first half of 2004-05 are:
Particulars
Sales (in ` '000):
Haryana
Rajasthan
Variable costs (as a % on sales):
Manufacturing
Selling
Specific fixed manufacturing expenses (in ` '000)

59

1,200
300

900
300

900
300

40
3
570

35
2
470

30
2
610

Ideal Classes Final CA Advanced Management Accounting Volume 1


You are required to:
1. Prepare a State-wise profit statement for the first half of 2004-05 using contribution approach.
Also offer your views on the contention of the management and opinion expressed by incharge
marketing division.
2. Prepare a product-wise profit statement for the same period using contribution approach.
3. Submit your well throughout recommendation as to which the product should be produced to
utilize idle capacity.
(Nov 1996)
Q.110. (Special Orders)
A Co. Ltd. manufactures several different styles of jewellery cases. Management estimates that
during the third quarter, the company will be operating at 80% of the normal capacity. Because the
company desires a higher utilization of plant capacity, the company will consider a special order.
The company has received special order inquiries from two companies. The first order is from JCP
Co. Ltd., which would like to market a jewellery case similar to one of A Co. Ltd.s jewellery cases. JCP
jewellery case would be marketed under JCPs own label. JCP Co. Ltd. has offered A Co. Ltd. ` 57.50
per jewellery case for 20,000 cases to be shipped by the last date of the quarter. The cost data for A
Co. Ltd. jewellery case that would be similar to the specifications of JCP special order are as follows:
(`)
Regular selling price per unit
90
Costs per unit
Raw Materials
25
Direct Labour
(0.5 hour @ ` 60)
30
Overhead
(0.25 machine hour @ ` 40)
10
Total Costs
65
According to the specifications provided by JCP Co. Ltd., the special order case requires less
expensive raw materials. Consequently the raw materials will only cost ` 22.50 per case.
Management has estimated that the remaining costs, labour time and machine time will be the
same as for A Co. Ltd. jewellery case. The second special order was submitted by K Co. Ltd. for 7,500
jewellery cases at ` 75 per case. These jewellery cases, like the JCP cases, would be marketed under
K label and have to be shipped by the last date of the quarter. However, the K Jewellery case is
different from any jewellery case in the A Co. Ltd. line. The estimated per unit cost of this case are as
follows:
Raw Materials
Direct Labour
Overhead
Total Costs

(0.5 hour @ ` 60)


(0.5 machine hour @ ` 40)

` 32.50
` 30.00
` 20.00
` 82.50

In addition A Co Ltd. will incur ` 15,000 in additional setup costs and will have to purchase a ` 25,000
special device to manufacture these cases, this device will be discarded once the special order is
completed.
The A Co. Ltd.s manufacturing capabilities are limited to the total machine hours available. The
plant capacity under normal operations is 90,000 machine hours per year or 7,500 machine hours
per month. The budgeted fixed overhead for the current year amounts to ` 21,60,000. All
manufacturing overhead costs are applied to production on the basis of machine hours at ` 40 per
hour. A Co. Ltd. will have the entire quarter to work on the special orders. Management does not
expect any repeat sales to be generated from either special order. Company precludes from
subcontracting any portion of an order, when special orders are not expected to generate repeat
sales.
Required: Should A Co. Ltd. accept either special order? Justify your answer and show the
calculations.
(Nov 2000)

60

Ideal Classes Final CA Advanced Management Accounting Volume 1


Q.111. (Special Orders)
Jolly Fabrics manufactures quality napkins at its unit in Tirupur. The unit has a capacity of 60,000
napkins per month. Present monthly production for April is 40,000 napkins. Costs incurred for
production are as below:
(` per unit)
6
2
4
12

Direct Material
Direct Labour
Manufacturing overhead
Total

No fixed cost
Fixed cost 75%
Variable 25%

The marketing costs per unit is ` 7 (` 5 is variable). Marketing costs include distribution costs and
customer service costs. Present selling price is ` 22.50 per unit.
Due to a strike at its existing napkin supplier, a hotel group has offered to buy 10,000 napkins from
Jolly Fabrics @ ` 11 per napkin for the month of June. No further sales to the hotel are anticipated.
Fixed manufacturing costs and marketing costs are tied to the 60,000 napkins. The acceptance of the
special order is not expected to affect the selling price to regular customers.
No marketing costs are involved in special order. Prepare:
1.
Budgeted income statement for June.
2.
Actual income statement under absorption costing for April.
3.
Should Jolly Fabrics accept the special order from the hotel or not?
(May 1990)
Q.112. (Continuation of a product)
Elec Ltd. is engaged in the manufacture of four products in its factory. The production and sales
volume is much lower than the normal volume and so there is a substantial unfavorable variance in
the recovery of overheads. The sales and cost data for a year are as under:
(` in Lakhs)
Products
Particulars
A
B
C
D
Total
Sales
400
500
200
100
1,200
Direct materials
64
70
32
7
173
Direct wages
88
105
60
18
271
Factory overheads
128
172
120
24
444
Selling & admn. Overheads
80
100
40
20
240
Total costs
360
447
252
69
1,128
Profit (Loss)
40
53
(52)
31
72
Unabsorbed overheads
48
Net Profit
24
50 per cent of the factory overheads is variable at normal operating volume and the variable selling
and administration overheads account for 5% of sales.
Of the total sales of Product C half of the volume is used in the market for application in which
Product D can be substituted. Thus if Product C is not available the sales of Product D can be
increased by ` 100 lacs without any change in the fixed selling expenses.
Of the total sale of Product C about 25% is sold in conjunction with Product A. The customers will
not be able to substitute Product D and so the sales of Product A will be reduced by 12.5% of the
present level if Product C is withdrawn.
In the event of total discontinuance of Product C, the fixed factory and selling and administration
overheads will be reduced by ` 20 lacs. Alternatively if the production and sales of Product C is
maintained to the extent of 25% of the present level as service to Product A, there will be a
reduction in the fixed costs to the extent of ` 10 lakhs.
You are required to:
a.
Prepare statement to show the financial implication of :

61

Ideal Classes Final CA Advanced Management Accounting Volume 1


i.
ii.
iii.
b.

Continuance of Product C
Total discontinuance of Product C
Continuance of Product C only as service to customers using Product A whose
business will otherwise be lost.
Make your recommendation on the course of action to be taken by the company with such
comments as you may like to offer.
(May 1985)

Q.113. (Shut Down or Continue)


Universe Ltd. manufactures 20,000 units of X in a year at its normal production capacity. The unit
cost as to variable costs and fixed costs at this level are ` 13 and ` 4 respectively.
Due to trade depression, it is expected that only 2,000 units of X can be sold during the next year.
The management plans to shut down the plant. The fixed costs for the next year then is expected to
be reduced to ` 33,000. Additional costs of plant shut-down are expected at ` 12,000 should the
plant be shut-down? What is the shut-down point?
(May 1996)
Q.114. (Make or Buy)
B Ltd produces and sells Bicycles. It also manufactures the chains for its Bicycles. It expects to
produce and sell 24,000 Bicycles during 1999-2000. It is considering an offer from an outside vendor
to supply any number of chains at ` 12 per chain.
The accountant of B Ltd. reports the following costs for producing 24,000 chains:
(`)
Cost
Cost per unit
Total cost
Direct Material
5.00
1,20,000
Direct Labour
4.00
96,000
Variable manufacturing Overhead
2.00
48,000
Inspection, set up etc.
1.00
24,000
Machine rent
1.00
24,000
allocated fixed overhead
1.25
30,000
14.25
3,42,000
The following additional information is available:
1.
Inspection, set up etc. vary with the number of batches in which the chains are produced.
Currently chains are being produced in the batch size of 2,000 units.
2.
Direct labour cost represents wages to four workers who are exclusively engaged in the
manufacturing of chains. These workers are in permanent capacity and cannot be retrenched.
3.
If B Ltd. procures all its chains from outside vendor, it will not require the machine which it has
hired for manufacturing chains.
Required:
I.
Assume that if B Ltd. purchases chains from outside vendor, the facility (including workers)
where the chains are currently manufactured will remain idle. Should B Ltd. accept the offer
from outside vendor at the anticipated production and sale volume of 24,000 units.
II.
Whether your decision in (i) will change if facilities can be used to upgrade the Bicycle which
will result in an incremental revenue of ` 22 per Bicycle. The variable cost for upgrading would
be ` 18 and tooling cost would be ` 16,000.
III.
Assume that facilities will be used as stated in (ii) above. Further assume that with better
planning B Ltd. will be able to manufacture chains in the batch sized of 4,000 units (instead of
2,000 units) if it decides to produce chains inside.
(Nov 1995)

62

Ideal Classes Final CA Advanced Management Accounting Volume 1


Q.115. (Accessories, whether to be provided)
Ze-Te-Fashions is a high-fashion womens garments manufacturer. It is planning to introduce a new
fashion garment in the market in the forthcoming Diwali Season. Four metres of cloth (material) are
required to layout the dress pattern. After cutting, some material remains that can be sold as a cutpiece. The left-over material can also be used to manufacture a matching cap & handbag.
Ze-Te expects to sell 2,500 dresses, if matching caps & handbags are not provided and 20% more, if
matching caps and handbags are made available. The market research indicates that the cap and / or
handbag cannot be sold independently, but only as accessories with the dress. The following
combination of sales is expected:
Complete sets of dress, cap and handbag
Dress and Cap only
Dress and handbag only
Dress only

68%
12%
9%
11%
100%

The material used in the dress costs ` 60 per metre. The cost of cutting the dress, if the cap and
handbag are not manufactured, is estimated at ` 20 a dress and the resulting remnants can be sold
for ` 5 for each dress cut out. If the cap and handbag are to be manufactured, it requires a more
delicate and skilful cutting and hence cutting cost will increase by ` 8 per dress. The selling prices
and the other costs to complete the three items, once they are cut, are as follows:
Dress
Cap
Handbag

Selling Price Per Unit


400.00
29.00
18.00

Other Costs Per Unit


48.00
6.50
3.00

Other costs per unit exclude the cost of material and cutting.
You are required to prepare a statement showing:
Should the company go in for caps and handbags along with dresses? Substantiate your answer.
(May 2001)

63

Ideal Classes Final CA Advanced Management Accounting Volume 1

Questions Section 5
(Theory)
Q.116. Relevant cost analysis helps in drawing the attention of managers to these elements of cost which
are relevant for the decision. Comment
Ans. Relevant costs are pertinent or valid costs for a decision. These bear upon or influence decision and
are directly related to the decisions to be made. These are critical to the decision, and have
significance for it. These are the cost which generally respond to managerial decision making, and
have significance in arriving at correct conclusions. These costs are capable of making a difference in
user-decisions and enter into a choice between alternative courses of action. In specific terms,
relevant costs for decisions are defined as expected future costs that will differ under alternatives.
Relevant costs are futuristic in nature. These are the costs that are expected to occur during the time
period covered by the decision.
Q.117. Briefly explain the concept of Opportunity Cost.
Ans. It is the cost of opportunity lost by the diversion of an input factor from one use to another. It arises
following the limited supply of an input resource or demand.
It is a measure of the benefit or opportunity foregone or lost. The opportunity cost of the value of
opportunity foregone is taken into consideration when alternatives are compared.
The introduction of opportunity cost concept is helpful to the management in making profitability
calculations when one or more of the inputs required by one or more of the alternative courses of
action is already available. These inputs may nevertheless have a cost and this is measured by the
sacrifice made by the alternative action chosen or the cost that is given up in order to make them
available for the current proposal.
Q.118. What are incremental costs and sunk cost ? Discuss
Ans. Incremental costs:
The difference in total cost between two alternatives or production level is an incremental cost. It is
synonymous to differential cost. Incremental cost arises due to change of the level of activity. The
change may be due to adding of a new product; change of channels of distribution, adding capacity
etc. Incremental costs are not necessarily variable in nature.
Sunk Cost:
Costs which do not change under given circumstances and do not play any role in decision making
process are known as sunk costs. They are historical costs incurred in the past. In other worlds, these
are the costs which have been incurred by a decision made in past and cannot be changed by any
decision made in the future. These costs are, however, best basis of predicting future cots.
Amortisation of past expenses is the clearest kind of sunk cost.
Q.119. What is meant by Incremental Revenue?
Ans.

Incremental revenue is the additional revenue that arise from the production or sale of a group of
additional units. It is one of the two basis concepts the other being incremental cost which go
together with differential cost analysis. Incremental cost in fact is the added cost due to change
either in the level of activity or in the nature of activity.

Q.120. Briefly explain the implications of replacement costs and historical costs in financial reporting.

64

Ideal Classes Final CA Advanced Management Accounting Volume 1


Ans.

Replacement costs and historical costs are two alternative methods of showing assets in financial
reporting.
Historical cost is the actual cost of an asset at the time of its acquisition. Replacement cost is the
cost to be incurred on an asset if it is replaced. These two cost concepts, on which valuation of
assets is based for financial reporting purposes, differs because of price variations over a period of
time. In case the price of an asset remains the same with the passage of time then historical cost
coincides with replacement cost.
Under financial reporting, i.e. in the balance sheet assets are recorded at their historical cost. When
prices rise substantially over a period of time, historical costs do not properly indicate the actual
costs. For managerial decisions, therefore these costs should properly be adjusted for price
changes. The distinction between historical costs and replacement cost is thus relevant when past
experience has to be considered as a guide to future costs for a proposed course of action.

Q.121. What is cost analysis? How it is useful in decision making?


Ans.

Cost analysis is the break up or classification of the aggregate costs into relevant types. Such an
analysis of cost is an essential pre-requisite of controlling costs and for decision making.
Identifiability of cost (i.e. direct cost) with units of products or operations is one such basis of cost
classification. The importance of distinguishing costs as direct or indirect lies in the fact that direct
costs of a product or an activity can be accurately allocated while indirect costs have to be
apportioned on the basis of certain assumptions. Thus it is helpful for management if costs are
classified on the basis of their Identifiability with the units of products, processes or work orders.
This is so because direct costs are controllable at the operational level whereas indirect costs are not
amenable to such control.
For the purpose of decision making and control, costs are distinguished on the basis of their
relevance to the different types of decisions and control functions. Thus, expenditure which has
taken place, is irrelevant in a situation & sunk cost. Cost incurred as a result of past decisions which
cannot be altered by another decision at a subsequent date is known as sunk cost. Thus, for
decisions with future implications, a sunk cost is an irrelevant cost. If a decision has to be made
whether to replace the existing plant, the book value of the existing plant is to be regarded as a sunk
cost as it is irrelevant to the question of its replacement. The decisive factor would be the difference
in income which will result from the installation of a new plant, and the expected rate of return on
the new investment.
Costs which are relevant are only to be taken into account and all such costs are analysed
accordingly. Present and future cash expenditure connected with a decision are the result of proper
cost analysis.

Q.122. Indicate the major areas of short-term decisions in which differential cost analysis is useful.
Ans. Cost information is required both for short-term and long-run managerial problems. Differential costs
are of particular use in short-term problems which are non-repetitive, one time, ad-hoc problems.
The following are the most common short-term problems and areas where differential cost analysis
may be deployed.
1.
Accept or Reject special offers.
2.
Make or Buy decisions.
3.
Sell or Process decisions.
4.
Reduce or Maintain price decisions.
5.
Add or Drop production decisions.
6.
Continue or Shut down decisions.

65

Ideal Classes Final CA Advanced Management Accounting Volume 1


Q.123. Distinguish between Cost reduction and Cost management:
Ans.

Cost reduction is the achievement of real and permanent in the unit cost of goods manufactured or
services rendered without impairing their suitable for the use intended or diminution in the quality
of the product. It use the techniques like value analysis, work study, standardization, simplification
etc. It is a continuous process of critical cost examination, analysis and challenges of established
standards. Each aspect of the business namely products, processes, methods, procedures is critically
examined and reviewed with a view to improving the efficiency and effectiveness so that cost are
reduced. It presumes the existence of concealed potential savings in norms or standards. It is a
corrective action.
Cost management is a broader concept. It aims at optimal utilization of resources to enhance the
operating income of the firm. It does not consider product attributes as given. It does not focus on
cost independent of revenue. Cost management system establishes linkage between cost and
revenues. It relates costs with product to have an insight into how various attributes generates
revenue and create demand on resources. It provides information to manage product attributes to
optimize resource utilization.
Traditional cost reduction systems focus on products, while cost management systems focus on
products, markets, and customers.

Q.124. What is CVP analysis and what purposes does it serve? Explain.
Ans. Profit per unit of a product depends on its selling price and cost of sales. Total profit depends on sales
volume which in turn depends inter-alia on selling price. By and large cost also depends on volume
of production. Thus, a close relationship exists between costs, volume and profit. Analysis of this
relationship opens up an interesting and useful field for the management accountant. Cost-volumeprofit analysis may be applied for profit planning, cost control, and decision making.
The following purposes are served by analysis of cost-volume-profit relationship:
1. To forecast profit fairly accurately.
2. To set up flexible budgets.
3. To evaluate performance for control.
4. To ascertain the effects of costs of changes in volume for market expansion or contraction.
5. To formulate price policies.
6. To know the amount of overhead costs that could be changed to productions costs at various
levels of operation.
Q.125. Discuss the four main limitations of break-even chart.
Ans. The main limitations of break-even chart are as follows :
1. The variable cost line need not necessarily be a straight line because of the possibility of
operation of law of increasing costs or law of decreasing returns.
2. Similarly the selling price will not be a constant factor. Any increase or decrease in output is
likely to have an influence on the selling price.
3. When a number of products are produced, separate break-even charts have to be drawn. This
poses a problem of apportionment of fixed expenses to each product.
4. Break-even charts ignore the capital employed in business which is one of the important guiding
factor in the determination of profitability.
5. The preparation of break-even chart presumes that costs can be reliably divided into fixed and
variable component. This is very difficult in practice.
6. The break-even chart presumes that production and sales will be synchronized at all points of
time or in other words, the entire production will be sold. This may not be true in practice.

66

Ideal Classes Final CA Advanced Management Accounting Volume 1


Q.126. Explain Profit/Volume Ratio.
Ans.

Profit volume ratio is the ratio of contribution denoting the difference between sales and variable
cost. Since in the short term fixed cost does not change, Profit/volume ratio also measures the rate
of change of profit due to change in the volume of sales.
Thus Profit/Volume ratio is expressed as Profit/Volume =

C
S

S-V
S

It is influenced by sales and variable or marginal cost. If the sales price increases without
corresponding increase in marginal cost the contribution increases and the Profit volume ratio
improves. Similarly if the marginal cost is reduced with sale price remaining same Profit/Volume
ratio improves.
The advantage of profit/volume ratio is that it can be used to measure profitability of each product
or group of them separately so that the necessity for continuance of such production can be
examined. It may also be used to measure the profitability of each production centre, process or
operation.
One fundamental property of Profit/Volume ratio is that it remains same at various levels of
operation and thus break even points, required selling prices to maintain profits at various levels etc.
can be easily calculated by suitable application of this ratio.
Q.127. Explain Breakeven point.
Ans. Breakeven point represents that volume of production where total cost equals total revenue resulting
into a no-profit no-loss situation. If output falls below that point, there is loss and if output exceeds
the point there is profit. Therefore at breakeven point:
Revenue = Total Cost
Sales = Fixed Cost + Variable Cost
Sales-variable Cost = Contribution = Fixed Cost
It can be concluded that at breakeven point the contribution earned just covers the fixed cost and at
levels below the point contribution earned is not adequate to match the fixed cost and at levels
above the point contribution earned more than recovers the fixed cost.
`

C
P

FC

Units
O
P is the breakeven point in the break even chart where OD and OC being the sales line and cost line
intersects. Loss result in the left side of i.e. before the breakeven point is reached and beyond P
profit starts to generate.

67

Ideal Classes Final CA Advanced Management Accounting Volume 1


Breakeven point has a wide use in the field of marginal costing and helps to decide the production
mix, fixation of price, to be taken in long term planning etc.
Q.128. Briefly discuss on curvilinear CVP analysis.
Ans.

In CVP analysis, the usual assumption is that the total sales line and variable cost line will have linear
relationship, that is, these lines will be straight lines. However, in actual practice it is unlikely to have
a linear relationship for two reasons, namely:
1. After the saturation point of existing demand the sales value may show a downward trend.
2. The average unit variable cost declines initially, reflecting the fact that, as output increases the
firm will be able to obtain bulk discounts on the purchase of raw materials and can also benefit
from division of labour. When the plant is operated at further higher levels of output, due to
bottlenecks and breakdowns the variable costs per unit will tend to increase. Thus the law of
increasing costs may operate and the variable cost per unit may increase after reaching a
particular level of output.
In such cases, the contribution will not increase in linear proportion on the phenomenon of
diminishing marginal productivity, the total cost line will not be straight, as assumed but will be of
curvilinear shape. This situation will give rise to two break even points. The optimum profit is earned
at the point where the distance between sales and total cost is the greatest.
In the diagram, A1, A2 and A3 are the three break-even points

Total revenue
A3

Total costs

Loss zone
A2
Amt
Profit zone
A1

Quantity
Q.129. Explain Margin of Safety.
Ans.

Margin of safety is the difference between the sales or production at a particular level of activity and
the break even sales or production. A large margin of safety indicates the soundness of the business
and correspondingly a small margin of business indicates a not too-sound position. Margin of safety
can be improved by lowering the fixed cost and variable costs, increasing the volumes of sales and
production, increasing the selling prices or changing the product mix resulting into a better overall
Profit/Volume ratio.
Margin of safety = Profit P/V ratio

Q.130. Explain Angle of Incidence.


Ans. It is the angle of intersection between the sales and the total cost lines. It indicates the profit earning
capacity of the concern at a certain level of sales production. The larger the angle of incidence the

68

Ideal Classes Final CA Advanced Management Accounting Volume 1


more is the profit earning capacity and vice versa. It also provides an indication as to what extent the
output and sales price may be varied to attain a desire level of profit. It gives an easy and clear idea
to the profitability under different levels of activities and also for different product mix and is a
simple visual aid to find out profit earning capacity without going in for any calculation.
Q.131. List out the assumptions of cost-volume-profit analysis.
Ans.

Q.132.

Ans.

The assumptions of cost-volume-profit analysis are as follows:


All variables remain constant per unit.
A single product or constant sales mix.
Fixed costs do not change.
Profits are calculated on variable cost basis.
Total costs and total revenues are linear functions of output.
The analysis applies to relevant range only.
Costs can be accurately divided into fixed and variable components.
The analysis applies only to short-term horizon.
Indicate any five circumstances under which you will permit to fix a price, which is less than the
marginal cost of the product.
Circumstances under which a firm may fix a price less than the marginal cost of the product are as
under
1. When goods are of perishable nature.
2. When the concern had already purchased huge quantities of raw materials and the prices of
these materials is falling considerably in the market.
3. When competitors are to be eliminated from the market.
4. When a new product is to be introduced in the market.
5. To obviate shut-down costs.
6. To push-up the sale of another highly profitable product.
7. To capture future market.
8. To capture foreign market.

Q. 133. Write notes on Key Factor


Ans. Every business aims to produce and sell unlimited units of the product manufactured by him. But it is
not possible due to some factor. There is always a factor which may limit the activity level of a firm.
Such a factor is known as the key factor. In most of the cases sales is the key factor. It determines
the volume of output to be produced. Sometime sales may not be the key factor but some other
factor such as labour; machine capacity; material; finance etc. which may not be available in requisite
quantity will be a key factor.
In other words, key factor is a factor that limits the quantum of activity of a firm at a particular time
or over a period of time. Key factor governs the decision how much to produce.
In case, sales being the key factor, the profitability of the product is measured by computing its profit
volume ratio. When any other factor is the key-factor, the most profitable product will be that which
would yield maximum contribution per unit of key factor. The profitability of any key-factor other
than sales can be ascertained by using the following formula:
Profitability (per unit) of key factor =
Contribution Key factor
Q. 134. Discuss the relationship between Angle of Incidence, break-even Level and Margin of Safety.

69

Ideal Classes Final CA Advanced Management Accounting Volume 1


Ans.

Angle of Incidence : It is the angle between total sales line and total cost line drawn in the case of
break-even chart. It provides useful information about the rate at which profits are being made. The
large the angle, the higher the rate of profit or vice-versa.
Break-even level : It is that level of sales (or production) at which the sales revenue exactly equals
total costs, both variable and fixed. In other words, it is level of activity at which the firm neither
earns a profit nor suffers a loss.
Margin of safety : It is the difference between total sales at break-even point. In other words margin
of safety is the amount of sales above the break-even point. If there is a fall in the sales to the
extent of margin of safety, the firm will not be in the red.
Relationship between Angle of Incidence, Break-even Level and Margin of Safety
1.

2.
3.
4.

If the break-even point is low and angle of incidence is large, the margin of safety is large and
the business enjoys financial stability. A low break-even point indicates that the business
could be run profitably even if there is a fall in sales, unless the sales are very low.
If the break-even point is low and angle of incidence is small, the conclusions are the same as
in 1 above except that the rate of profit earning capacity is not so high as in 1.
If the break-even point is high and angle of incidence is small, the margin of safety is low. The
business is very vulnerable, even a small reduction in activity may result in a loss.
If the break-even point is high and angle of incidence is large, this shows that the margin of
safety is low. The business is likely to incur losses through a small reduction in activity.
However, after the break-even point, the business makes the profit at a high rate.

Q. 135. Mention any four important factors to be considered in Marginal Costing Decisions.
Ans. Important factors to be considered in Marginal Costing Decisions are as follows :
1. Whether the product or production makes a contribution.
2. In the selection of alternatives, additional fixed costs if any should be considered.
3. The continuity of demand after expansion and its impact on selling price are to be considered.
4. Non-cost factors such as the need to keep labour force intact and government attitude are also
to be taken into account.
Q. 136. List out the assumptions of break-even analysis
Ans.

The assumptions underlying break even analysis are as below :


1. All costs can be easily classified into fixed and variable components.
2. Both revenue and cost functions are linear over the range of activity under consideration .
3. Prices of output and input remain unchanged.
4. Productivity of the factors of production will remain the same.
5. The state of technology and the process of production will not change.
6. There will be no significant change in the levels of inventory.
7. The company manufactures a single product.
8. In the case of a multi-product company, the sales mix will remain unchanged.

Q. 137. What are the limitations of marginal costing? Explain.


Ans.

1.

Marginal Costing assumes that all costs can be classified into fixed and variable. But it is not so,
as there are costs which are neither fixed or variable. For example, various amenities provided
to workers may have no relation either to volume of production or time factor.
2. Contribution of a product itself is not a guide for optimum profitability unless it is linked with the
key factor.
3. Marginal Costing ignores time factor and investment. For example, the marginal cost of two jobs
may be the same but the time taken for their completion and the cost of machines used may
differ.

70

Ideal Classes Final CA Advanced Management Accounting Volume 1


4. The overheads of fixed nature cannot altogether be excluded from particular contracts while
valuing work-in-progress. In order to show the correct position, fixed overheads have to be
included in work-in-progress.
5. In the long run, the selling prices should be based on total cost i.e. inclusive of fixed cost also. In
the short run or in special situations when a product is sold below the total cost, customers may
insist on the continuation of reduced prices forever which may not be possible in all cases.
6. The main assumption regarding behavior of costs are not true. The variable costs do not remain
constant per unit of output. There may be changes in the prices of raw materials, wage rates
etc., after a certain level of output has been reached due to shortage of skilled labour,
concessions of bulk purchases etc. Similarly, the fixed costs does not remain static. They may
change from one period to another. For example, salaries bill may go up because of annual
increments or due to change in pay rate etc.
Q. 138. Distinguish between absorption costing and marginal costing.
(For answer, kindly refer classroom notes)
Q.139. The use of the Absorption costing method in management decision making process leads to
anomalies. Discuss.
Ans.

Absorption Costing has been defined by the Institute of Cost and Management Accounts, London as
The practice of charging all costs, both variable and fixed, to operation, processes or products.
The ordinary system of costing in which all overheads are apportioned over various units of cost,
without making a distinction between fixed and variable expenses, is absorption costing. This means
that the value of inventory is based on all costs, fixed as well as variable. But fixed costs are period
costs and hence have no direct relation with output. Any basis on which these costs are allocated or
apportioned to production departments or absorbed by output are generally arbitrary. As such it
leads to anomalous situations and the allocation of fixed costs to products vitiates the correctness of
units cost. It is quite misleading in management of decision, particularly when a product is to be
dropped or make or buy decision is to be taken. In such cases, a decision based on total cost will not
be correct. In absorption costing, profit becomes a function of production instead of sales. It is thus
obvious that use of the absorption costing method in management decision making process leads to
anomalies.

Q. 140. Distinguish between Cost indifference point and Break-even point


Ans.

Cost indifference point : It is the point at which total cost lines under the two alternatives intersect
each other. Cost indifference point is calculated as under:
Difference in fixed costs saving in variable cost
Break-even point: It is the point where the total cost line and total revenue line for a particular
alternative intersect each other. Break-even point is calculated as under:
Fixed costs Contribution per unit or the Fixed costs PV ratio.
The following are the main point of distinction between cost indifference point and break-even
point.
1. The cost indifference point is the activity level at which total cost under two alternatives are
equal. Whereas break-even point is the activity level at which the total revenue form a product
mix is equal to its cost.
2. Cost indifference point is used to choose between two alternative processes for achieving the
same objective. The choice depends on the estimated activity level. Breakeven point is used for
profit planning.

71

S-ar putea să vă placă și